AAOS Recon 2013

You might also like

You are on page 1of 96

Adult Reconstructive Surgery

of the Hip and knee


Self-Assessment Examination 2013

ANSWER Book
All rights reserved. No part of Adult Reconstructive Surgery of the Hip and Knee Self-Assessment
Examination may be reproduced, stored in a retrieval system, or transmitted in any form or by any means
(electronic, mechanical, photocopying, recording, or otherwise) without the prior written permission of
the publisher.

Published January 2013


American Academy of Orthopaedic Surgeons
6300 North River Road
Rosemont, IL 60018

Copyright  2013 by the American Academy of Orthopaedic Surgeons

Requests for permission to reproduce any part of the work should be mailed to:
Attention: Examinations Department
American Academy of Orthopaedic Surgeons
6300 North River Road
Rosemont, IL 60018

Printed in the USA

 2013 American Academy of Orthopaedic Surgeons 2013 Adult Reconstructive Surgery of the Hip and Knee Self-Assessment Examination

© 2013 American Academy of Orthopaedic Surgeons 2013 Adult Reconstructive Surgery of the Hip and Knee Self-Assessment Examination
Dear Colleague,

Thank you for choosing the 2013 Adult Reconstructive Surgery of the Hip and Knee Self-
Assessment Examination. This examination was developed for orthopaedic surgeons in clinical
practice with subspecialty interest in hip and knee arthroplasty.

This examination features several types of questions. Basic science questions are intended to test
knowledge that is clinically relevant to reconstructive surgery of the hip and knee. Recall
questions will assess your knowledge of specific information that is important in clinical
practice. Questions that require your interpretation of diagnostic information or imaging studies
assess your ability to analyze pertinent positive or negative findings. Problem-solving questions
present clinical scenarios for which an assessment based on your clinical judgment of an
evaluation and/or treatment is required; several questions in a series may relate to one clinical
scenario.

This year’s examination covers many issues encountered in hip and knee arthroplasty practice,
with a focus on seven major topics. Infection prevention, infection treatment, thromboembolism,
pain issues, bearing surfaces, osteolysis, and hip instability are emphasized in this examination.

This examination’s questions were written and edited by members of the AAOS Hip, Knee and
Adult Reconstruction Evaluation Committee through a peer-review process intended to provide
objective questions that assess knowledge and an educational resource on clinical issues in hip
and knee arthroplasty. As committee chair, I would like to thank the committee members and
AAOS staff for their work in creating this examination, and thank you again for your interest in
taking the examination. I hope this tool is as helpful as it was intended to be throughout its
development.

Sincerely,

Michael D. Ries, MD
Chair, Hip, Knee and Adult Reconstruction Evaluation Committee

© 2013 American Academy of Orthopaedic Surgeons 2013 Adult Reconstructive Surgery of the Hip and Knee Self-Assessment Examination
 2013 American Academy of Orthopaedic Surgeons 2013 Adult Reconstructive Surgery of the Hip and Knee Self-Assessment Examination
ACCREDITATION
The American Academy of Orthopaedic Surgeons is accredited by the Accreditation Council for
Continuing Medical Education to provide continuing medical education for physicians.

AMA PRA CREDIT


U. S. Physicians: The American Academy of Orthopaedic Surgeons designates this enduring
material for a maximum of 10 AMA PRA Category 1 Credits™. Physicians should claim only
the credit commensurate with the extent of their participation in the activity.

International Physicians: AMA PRA credit may only be claimed by, and awarded to,
physicians, defined by the AMA as individuals who have completed an allopathic (MD),
osteopathic (DO), or an equivalent medical degree from another country.

Allied Health Professionals: The American Academy of Orthopaedic Surgeons is not accredited to
offer credit for nurses and other allied health professionals. To determine if activities offering AMA PRA
Category 1 Credits™ are acceptable for your licensing or certification needs, please contact the relevant
organization(s) directly.

Independent Study (previously called Self-Scored) Examination Participants: The AAOS no


longer awards CME credit for printed independent study self-assessment examinations as a result
of new requirements from the AMA’s Physician Recognition Award Program.

EDUCATIONAL OBJECTIVES
As a result of taking the 2013 Adult Reconstructive Surgery of the Hip and Knee Self-
Assessment Examination, I am able to
• identify important factors in the evaluation of periprosthetic infection.
• diagnose and treat acute and chronic infection after total hip or knee arthroplasty.
• apply the AAOS guidelines for prevention and management of thromboembolic
complications.
• evaluate causes of pain after total hip or knee arthroplasty.
• determine factors contributing to wear of different bearing surfaces.
• evaluate and treat periprosthetic osteolysis.
• identify factors which can lead to hip instability and appropriate treatment methods.

INQUIRIES
Questions regarding completing this CME activity or other comments may be sent to
exams@aaos.org or write to: Attention: Examinations Department, American Academy of
Orthopaedic Surgeons, 6300 North River Road, Rosemont, IL 60018.

RELEASE DATE: January 2013


EXPIRATION DATE: December 31, 2015.
No CME credit will be awarded for this activity after December 31, 2015

 2013 American Academy of Orthopaedic Surgeons 2013 Adult Reconstructive Surgery of the Hip and Knee Self-Assessment Examination

© 2013 American Academy of Orthopaedic Surgeons 2013 Adult Reconstructive Surgery of the Hip and Knee Self-Assessment Examination
Produced by the American Academy of Orthopaedic Surgeons
Hip, Knee and Adult Reconstruction Evaluation Committee of the
Central Evaluation Committee

Michael D. Ries, MD, Editor in Chief


Professor of Orthopaedic Surgery, University of California, San Francisco; Chief of
Arthroplasty, University of California, San Francisco, California

Contributors
B. Sonny Bal, MD
Associate Professor, Department of Orthopaedic Surgery, University of Missouri School of
Medicine, Columbia, Missouri
James P. Crutcher Jr, MD
Attending Surgeon, Swedish Orthopedic Institute; Proliance Surgeons, Inc; Immediate Past
Chief, Department of Orthopaedics, Swedish Medical Center, Seattle, Washington
Jonathan P. Garino, MD
Clinical Professor of Orthopedic Surgery, Lankenau Institute for Medical Research,
Villanova, Pennsylvania
Michael H. Huo, MD
Professor of Orthopedic Surgery, University of Texas Southwestern School of Medicine,
Dallas, Texas
Stephen J. Incavo, MD
Section Chief, Adult Reconstructive Surgery, Program Director, Adult Hip and Knee
Reconstructive Surgery Fellowship, Methodist Center for Orthopaedic Surgery; Professor
of Clinical Orthopaedic Surgery, Weill Cornell Medical College of Cornell University,
Houston, Texas
Gwo-Chin Lee, MD
Assistant Professor of Orthopaedic Surgery Fellowship Director- Adult Reconstruction
Department of Orthopaedic Surgery, University of Pennsylvania, Philadelphia,
Pennsylvania
Russell D. Meldrum, MD
Adjunct Professor of Orthopaedic Surgery, Indiana University School of Medicine,
Indianapolis, Indiana
Lalit Puri, MD
Associate Professor, Department of Orthopaedic Surgery, Northwestern University
Feinberg School of Medicine Northwestern Memorial Hospital, Chicago, Illinois
James J. Purtill, MD
Associate Professor, Vice Chairman, Department of Orthopaedic Surgery, Thomas
Jefferson University, Philadelphia, Pennsylvania
Mark J. Spangehl, MD
Assistant Professor, Mayo Medical School, Department of Orthopaedic Surgery, Program
Director, Adult Reconstruction Fellowship, Mayo Clinic Arizona, Phoenix, Arizona
James B. Stiehl, MD
Associate Clinical Professor of Orthopaedic Surgery, Medical College of Wisconsin,
Milwaukee, Wisconsin

 2013 American Academy of Orthopaedic Surgeons 2013 Adult Reconstructive Surgery of the Hip and Knee Self-Assessment Examination

© 2013 American Academy of Orthopaedic Surgeons 2013 Adult Reconstructive Surgery of the Hip and Knee Self-Assessment Examination
ACKNOWLEDGMENTS

Medical Editing
Brenda Moss Feinberg, ELS, AAOS Examinations Consultant

Graphic Design and Digital Media Specialist


John Cisco, AAOS Examinations Consultant

American Academy of Orthopaedic Surgeons


Constance M. Filling, Chief Education Officer

Department of Electronic Media, Examinations, CME Course Operations, and Practice Management
Howard Mevis, Director
Laura Hruska, M. Ed., Manager, Examinations
Marcie L. Lampert, Senior Coordinator, Examinations
Anna M. Scheer, Senior Coordinator, Examinations
Denise Plasky, Assistant, Examinations
Irene Bogdal, Administrative Assistant, Examinations

 2013 American Academy of Orthopaedic Surgeons 2013 Adult Reconstructive Surgery of the Hip and Knee Self-Assessment Examination

© 2013 American Academy of Orthopaedic Surgeons 2013 Adult Reconstructive Surgery of the Hip and Knee Self-Assessment Examination
Disclosure Information for 2013 Adult Reconstructive Surgery of the Hip and Knee Self-
Assessment Examination

• Michael D. Ries, MD (Chair): 1 (Smith & Nephew;);3B (Smith & Nephew; Stryker;);4
(OrthAlign;);8 (The Knee;);9 (Foundation for the Advancement of Research in Medicine;);
Submitted on: 05/10/2012. *
• B. Sonny Bal, MD (Member): 1 (Zimmer);3B (Zimmer);3C (OMNI Amedica Medtronic
ConforMIS);4 (Amedica);8 (Journal of Arthroplasty Clinical Orthopaedics and Related Research
OrthoMind Bonesmart.org); Submitted on: 05/01/2012. *
• James P. Crutcher Jr, MD (Member): 2 (Stryker);3B (Stryker);4 (Exactech, Inc Mako Surgical);8
(Journal of Arthroplasty); Submitted on: 04/11/2012. *
• Jonathan P. Garino, MD (Member): 1 (Smith & Nephew);2 (Smith & Nephew);3B (Smith &
Nephew; ; DePuy, A Johnson & Johnson Company);5 (Zimmer);8 (Knee; seminars in arthroplasty;
techniques in arthroplasty; BMC Musculoskeletal Disorders);9 (Pennsylvania orthopedic society);
Submitted on: 06/02/2012. *
• Michael H. Huo, MD (Member): 2 (Jassen; Cadence Pharmaceutical);3B (DePuy);9 (American
Academy of Orthopedic Surgeons (Committee on Evaluation: self-assessment examination));
Submitted on: 04/16/2012. *
• Stephen J. Incavo, MD (Member): 1 (Wright Medical Technology, Inc.; Stryker, Innomed);2
(Stryker; Wright Medical Technology, Inc.);3B (Wright Medical Technology, Inc., Stryker);4
(Wright Medical Technology, Inc.; Stryker; Nimbic Systems);5 (Stryker; Surgical Monitoring
Associates, Inc.; Surgical Synergies; Synthes);8 (Journal of Arthroplasty; Knee; Clinical
Orthopaedics and Related Research: Journal of Bone and Joint Surgery: The Knee; Journal of
Orthopaedic Research);9 (AAOS; American Association of Hip and Knee Surgeons; Knee Society;
Hip Society); Submitted on: 04/09/2012. *
• Gwo-Chin Lee, MD (Member): 8 (Clinical Orthopaedics and Related Research; Journal of
Arthroplasty; Orthopedics; SLACK Incorporated); Submitted on: 06/01/2012. *
• Russell D. Meldrum, MD (Member): (n) Submitted on: 06/01/2012. *
• Lalit Puri, MD (Member): 1 (Innomed; Stryker);3B (Stryker Salient Surgical; Kinamed); Submitted
on: 04/19/2012. *
• James J. Purtill, MD (Member): 8 (Journal of Arthroplasty); Submitted on: 05/31/2012. *
• Mark J. Spangehl, MD (Member): 5 (Stryker); 9 (AAOS); Submitted on: 05/19/2012. *
• James B. Stiehl, MD (Member): 1 (Zimmer; Innomed);2 (Blue Orthopaedics Computer Company;
Zimmer);3B (Zimmer Blue Orthpaedics Computer Company);3C (Exactech, Inc);4 (Blue
Orthopaedics Computer Company Traumis Inc, Technology Company);8 (Knee; Journal of
Arthroplasty); Submitted on: 06/05/2012. *

American Academy of Orthopaedic Surgeons Staff


• Howard Mevis (Rosemont, IL): 4 (GE Healthcare; 3M; Novartis); 9 (Orthopaedic Learning Center);
Submitted on: 09/05/2012. *
• Laura Hruska (Rosemont, IL): (n) Submitted on: 04/30/2012. *
• Marcie Lampert (Staff Liaison): (n) Submitted on: 09/07/2012. *

* Disclosure Items Answered: (n) = Respondent answered 'No' to all items indicating no conflicts.
1= Royalties from a company or supplier; 2= Speakers bureau/paid presentations for a company or supplier; 3A=
Paid employee for a company or supplier; 3B= Paid consultant for a company or supplier; 3C= Unpaid consultant
for a company or supplier; 4= Stock or stock options in a company or supplier; 5= Research support from a company
or supplier as a PI; 6= Other financial or material support from a company or supplier; 7= Royalties, financial or
material support from publishers; 8= Medical/Orthopaedic publications editorial/governing board; 9= Board
member/committee appointments for a society.

 2013 American Academy of Orthopaedic Surgeons 2013 Adult Reconstructive Surgery of the Hip and Knee Self-Assessment Examination
Disclaimer
The material presented in this self-assessment examination has been made available by the
American Academy of Orthopaedic Surgeons for educational purposes only. This material is not
intended to present the only, or necessarily the best, methods or procedures for the medical
situations discussed, but rather is intended to represent an approach, view, statement, or opinion
of the author(s) or producer(s), that may be helpful to others who face similar situations.

Some drugs or medical devices demonstrated in Academy courses or described in Academy print
or electronic publications have not been cleared by the Food and Drug Administration (FDA) or
have been cleared for specific uses only. The FDA has stated that it is the responsibility of the
physician to determine the FDA clearance status of each drug or device he or she wishes to use in
clinical practice.

Furthermore, any statements about commercial products are solely the opinion(s) of the author(s)
and do not represent an Academy endorsement or evaluation of these products. These statements
may not be used in advertising or for any commercial purpose.

 2013 American Academy of Orthopaedic Surgeons 2013 Adult Reconstructive Surgery of the Hip and Knee Self-Assessment Examination
2013 Adult Reconstructive Surgery of the Hip and Knee Self-Assessment Examination Answer Book • 11

Question 1
Cementless knee replacements are associated with

1. increased risk for revision.


2. increased wear of the polyethylene insert.
3. increased infection risk.
4. significantly higher Knee Society Scores (KSS).

PREFERRED RESPONSE: 1

DISCUSSION
Many published series comparing cementless to cemented knee replacements show no difference in KSS
or infection risk, but data in several registries demonstrate a significant increase in the revision rates for
cementless knees. Although no difference in poly wear occurred, loosening of the tibial component was
the most common cause of failure.

RECOMMENDED READINGS
Paxton EW, Inacio M, Slipchenko T, Fithian DC. The kaiser permanente national total joint replacement
registry. Perm J. 2008 Summer;12(3):12-6. PubMed PMID: 21331204.
Gandhi R, Tsvetkov D, Davey JR, Mahomed NN. Survival and clinical function of cemented and
uncemented prostheses in total knee replacement: a meta-analysis. J Bone Joint Surg Br. 2009
Jul;91(7):889-95. Review. PubMed PMID: 19567852.

© 2013 American Academy of Orthopaedic Surgeons 2013 Adult Reconstructive Surgery of the Hip and Knee Self-Assessment Examination
12 • American Academy of Orthopaedic Surgeons

Figure 2a Figure 2b

CLINICAL SITUATION FOR QUESTIONS 2 THROUGH 4


Figures 2a and 2b are the radiographs of a 56-year-old woman who has groin pain and decreased function
15 months after a total hip replacement using the posterolateral approach. Findings from the workup for
infection are negative and physical examination localizes pain to the hip joint. Records show the patient
underwent total hip surgery with a metal-on-metal bearing.

The patient undergoes cup revision to a more optimal position using a 36-mm metal-polyethylene bearing.
Abductor muscle damage from gross metal debris and inflammation are encountered during surgery.
After cup revision, repair of abductors, and hip brace use for several weeks, the patient has no more pain.
Several months after surgery, the patent returns with a history of multiple hip dislocations that have proven
refractory to treatment including hip precautions, bracing, and exercise. Radiographs show perfectly
positioned components; the patient has no pain, and examination under anesthesia shows hip instability.

Question 2
Before recommending revision total hip arthroplasty, what other step(s) should be included in the workup?

1. Aspiration of the hip joint and diagnostic injection of an anesthetic


2. Draw an erythrocyte sedimentation rate (ESR) and C-reactive protein (CRP)
3. Three-phase bone scan of the hip
4. Lumbar spine radiographs

PREFERRED RESPONSE: 2

Question 3
Before this patient’s most recent revision surgery, her symptoms were most likely related to

1. systemic metal ion debris.


2. component malposition.
3. leg length inequality.
4. Head-neck taper corrosion.

PREFERRED RESPONSE: 2
© 2013 American Academy of Orthopaedic Surgeons 2013 Adult Reconstructive Surgery of the Hip and Knee Self-Assessment Examination
2013 Adult Reconstructive Surgery of the Hip and Knee Self-Assessment Examination Answer Book • 13

Question 4
After revision surgery, this patient’s total hip remains unstable and unresponsive to nonsurgical treatment.
What is the most appropriate surgical option?

1. Trochanteric advancement
2. Revision to a constrained polyethylene liner
3. Revision to the largest head size and increase leg length
4. Resection with repeat abductor repair, with staged reimplantation

PREFERRED RESPONSE: 2

DISCUSSION FOR QUESTIONS 2 THROUGH 4


The differential diagnosis of pain after a total hip arthroplasty encompasses a number of etiologies,
but the question is directed to a basic and essential part of the workup (ie, definitively considering and
ruling in or out the possibility of deep sepsis). Radiographs may point to other, obvious sources of
pain, but the orthopaedic surgeon must not overlook the possibility that deep sepsis is the predominant
cause of the symptoms. Accordingly, ESR and CRP are logical next steps in the workup in this clinical
scenario. Radiographs show increased anteversion of the metal socket, and pain etiologies can include
psoas irritation, hip instability, or adverse tissue reaction to metal debris generated by suboptimal implant
position leading to higher bearing contact stresses and/or impingement. Once other common etiologies
of hip pain have been excluded such as deep infection or lumbar pathology, the most likely cause of hip
symptoms should be considered. Here the evidence points to a malpositioned acetabular component.
Systemic ion dissemination may occur in this patient but will not produce hip pain. Head-neck taper
corrosion can generate metallic debris, but a more likely source of local metallic debris is edge loading
or impingement of the metal-metal bearing. Leg length inequality can be distressing to a patient but
will usually not result in hip pain. Component malposition is the best answer. Among the spectrum of
clinical presentations following failed metal-metal total hip replacements, abductor damage from localized
inflammation is one finding that can lead to hip instability. A reasonable treatment option is to repair the
abductors as best as possible, with augmentation of soft-tissue repair using graft tissue, a large-diameter
femoral head, and a constrained polyethylene liner. This is a challenging clinical scenario because chronic
hip instability with deficient abductors is difficult to control and is an indication for the use of constrained
components. Revision to a larger head and increased leg lengths will not address the underlying cause of
instability. Hip resection is not necessary because this is not a septic total hip.

© 2013 American Academy of Orthopaedic Surgeons 2013 Adult Reconstructive Surgery of the Hip and Knee Self-Assessment Examination
14 • American Academy of Orthopaedic Surgeons

RECOMMENDED READINGS FOR QUESTIONS 2 THROUGH 4


Parvizi J, Azzam K, Ghanem E, Austin MS, Rothman RH. Periprosthetic infection due to resistant
staphylococci: serious problems on the horizon. Clin Orthop Relat Res. 2009 Jul;467(7):1732-9. Epub
2009 May 1. PubMed PMID: 19408061.
Shukla SK, Ward JP, Jacofsky MC, Sporer SM, Paprosky WG, Della Valle CJ. Perioperative testing for
persistent sepsis following resection arthroplasty of the hip for periprosthetic infection. J Arthroplasty.
2010 Sep;25(6 Suppl):87-91. PubMed PMID: 20732621.
Browne JA, Bechtold CD, Berry DJ, Hanssen AD, Lewallen DG. Failed metal-on-metal hip arthroplasties:
a spectrum of clinical presentations and operative findings. Clin Orthop Relat Res. 2010 Sep;468(9):2313-
20. PubMed PMID: 20559767.
Onda K, Nagoya S, Kaya M, Yamashita T. Cup-neck impingement due to the malposition of the implant
as a possible mechanism for metallosis in metal-on-metal total hip arthroplasty. Orthopedics. 2008
Apr;31(4):396. PubMed PMID: 19292272.
Killampalli VV, Reading AD. Late instability of bilateral metal on metal hip resurfacings due to
progressive local tissue effects. Hip Int. 2009 Jul-Sep;19(3):287-91. PubMed PMID: 19876887.
Sikes CV, Lai LP, Schreiber M, Mont MA, Jinnah RH, Seyler TM. Instability after total hip arthroplasty:
treatment with large femoral heads vs constrained liners. J Arthroplasty. 2008 Oct;23(7 Suppl):59-63.
Review. PubMed PMID: 18922375.

END OF SERIES

Question 5
A 67-year-old active man returns for routine follow up 12 years after hip replacement. He has no hip pain.
Radiographs revealed a well-circumscribed osteolytic lesion around a single acetabular screw. All hip
components were perfectly positioned. Six months later, comparison radiographs show an increase in the
size of the osteolytic lesion. A CT scan shows a well-described lesion that is 3 cm at its largest diameter
and is localized around 1 screw hole with an eccentric femoral head. What treatment is appropriate,
assuming well-fixed cementless total hip components exist?

1. Revision of the polyethylene liner, removal of the screw, and debridement of the osteolytic
lesion with or without bone grafting
2. Revision of the acetabular component to a newer design without screws
3. Removal of the screw, revision of the polyethylene liner, and stem cell injection into the lytic
lesion
4. Removal of the offending screw from the metal socket and placement of a new polyethylene
liner in the existing socket

PREFERRED RESPONSE: 1

© 2013 American Academy of Orthopaedic Surgeons 2013 Adult Reconstructive Surgery of the Hip and Knee Self-Assessment Examination
2013 Adult Reconstructive Surgery of the Hip and Knee Self-Assessment Examination Answer Book • 15

DISCUSSION
With a well-fixed acetabular metal shell and a localized osteolytic lesion, good outcomes can be expected
with liner revision in this clinical scenario with retention of the metal socket, assuming no damage to the
components or other unexpected findings during revision surgery. Here, complete cup revision is not
warranted considering the appropriate implant position. Beaule and associates reviewed 83 consecutive
patients (90 hips) in which a well-fixed acetabular component was retained in clinical scenarios such
as the one described; no hip showed recurrence or expansion of periacetabular osteolytic lesions. If the
metal cup is unstable, or if the osteolytic lesion is not amenable to debridement through the screw hole,
acetabular component revision may be indicated.

RECOMMENDED READINGS
Beaule PE, Le Duff MJ, Dorey FJ, Amstutz HC. Fate of cementless acetabular components retained
during revision total hip arthroplasty. J Bone Joint Surg Am. 2003 Dec;85-A(12):2288-93. PubMed PMID:
14668496.
Mall NA, Nunley RM, Smith KE, Maloney WJ, Clohisy JC, Barrack RL. The fate of grafting acetabular
defects during revision total hip arthroplasty. Clin Orthop Relat Res. 2010 Dec;468(12):3286-94. PubMed
PMID: 20577842.

© 2013 American Academy of Orthopaedic Surgeons 2013 Adult Reconstructive Surgery of the Hip and Knee Self-Assessment Examination
16 • American Academy of Orthopaedic Surgeons

Figure 6a Figure 6b Figure 6c Figure 6d

Question 6
A 70-year-old man underwent removal of an infected total hip arthroplasty (THA) and insertion of an
articulating antibiotic-loaded spacer to treat a deep periprosthetic hip infection. He was in a nursing
home receiving intravenous antibiotics 3 weeks after surgery when he tripped and fell. Examination
shows swelling in the mid and distal thigh, intact skin and neurovascular structures, and severe pain with
knee or hip movement. Radiographs of the femur are shown in Figures 6a through 6d. What is the most
appropriate treatment for the fracture below the implant?

1. Balanced traction to address concern regarding persistent infection with reoperation


2. Open reduction and internal fixation of the fracture with a lateral plate and screws
3. Removal of the articulating spacer and revision to a longer-stem antibiotic-loaded articulating
spacer
4. Removal of the articulating spacer and reimplantation using a long-stem fluted uncemented
hip replacement

PREFERRED RESPONSE: 2

DISCUSSION
This patient has a type C periprosthetic femoral fracture. The articulating spacer is not involved in the
fracture, which is well distal to the implant. The most appropriate treatment is open reduction and internal
fixation of the fracture. Traction is not appropriate for this fracture because it can be treated surgically
despite the history of previous hip infection. Traction would also be needed for at least 5 weeks to delay
surgical treatment of the periprosthetic fracture until the time of second-stage revision THA. The fracture
is fairly distal and revision to a longer antibiotic-loaded implant or uncemented stem is not suitable for
this fracture pattern because it extends well past the isthmus. A femoral stem in the distal fragment would
provide little stability for the fracture. Additionally, removing the articulating spacer and reimplantation
using a long-stem fluted uncemented hip replacement is not appropriate because it would be premature to
reimplant this man’s hip while he is still receiving treatment for his deep-hip infection.

© 2013 American Academy of Orthopaedic Surgeons 2013 Adult Reconstructive Surgery of the Hip and Knee Self-Assessment Examination
2013 Adult Reconstructive Surgery of the Hip and Knee Self-Assessment Examination Answer Book • 17

RECOMMENDED READINGS
Laurer HL, Wutzler S, Possner S, Geiger EV, El Saman A, Marzi I, Frank J. Outcome after operative
treatment of Vancouver type B1 and C periprosthetic femoral fractures: open reduction and internal
fixation versus revision arthroplasty. Arch Orthop Trauma Surg. 2011 Jul;131(7):983-9. Epub 2011 Feb 18.
PubMed PMID: 21331551.
Haddad FS, Duncan CP, Berry DJ, Lewallen DG, Gross AE, Chandler HP. Periprosthetic femoral fractures
around well-fixed implants: use of cortical onlay allografts with or without a plate. J Bone Joint Surg Am.
2002 Jun;84-A(6):945-50. PubMed PMID: 12063328.

Question 7
What antithrombotic agent is a selective factor I0a inhibitor?

1. Warfarin
2. Low-molecular-weight heparin
3. Rivaroxaban
4. Aspirin

PREFERRED RESPONSE: 3

DISCUSSION
Rivaroxaban is a selective factor I0a inhibitor. Aspirin is a cyclooxygenase inhibitor. Low-molecular-
weight heparin is a nonspecific anticoagulant. Warfarin is a vitamin K antagonist and reduces production
of clotting factors II, VII, IX, and X.

RECOMMENDED READINGS
Haas SB, Barrack RL, Westrich G, Lachiewicz PF. Venous thromboembolic disease after total hip and
knee arthroplasty. J Bone Joint Surg Am. 2008 Dec;90(12):2764-80. PubMed PMID: 19047723.
Freedman KB, Brookenthal KR, Fitzgerald RH Jr, Williams S, Lonner JH. A meta-analysis of
thromboembolic prophylaxis following elective total hip arthroplasty. J Bone Joint Surg Am. 2000
Jul;82-A(7):929-38. PubMed PMID: 10901307.
Turpie AG, Lassen MR, Davidson BL, Bauer KA, Gent M, Kwong LM, Cushner FD, Lotke PA,
Berkowitz SD, Bandel TJ, Benson A, Misselwitz F, Fisher WD; RECORD4 Investigators. Rivaroxaban
versus enoxaparin for thromboprophylaxis after total knee arthroplasty (RECORD4): a randomised trial.
Lancet. 2009 May 16;373(9676):1673-80. Epub 2009 May 4. PubMed PMID: 19411100.

© 2013 American Academy of Orthopaedic Surgeons 2013 Adult Reconstructive Surgery of the Hip and Knee Self-Assessment Examination
18 • American Academy of Orthopaedic Surgeons

Question 8
What is the difference between annealed (below the melting temperature) and remelted highly crossed-
linked polyethelyne?

1. Annealing results in lower potential for oxidation in vivo.


2. Annealing results in less change to mechanical properties and strength compared to remelting.
3. Remelting of polyethylene eliminates the potential for oxidation.
4. Remelting of the polyethylene removes the remaining free radicals and makes the
polyethylene stronger.

PREFERRED RESPONSE: 2

DISCUSSION
Polyethylene is remelted to remove free radicals that formed during the radiation process for cross-linking.
The disadvantage of remelting polyethylene is that it reduces the mechanical properties of the material.
Annealing of polyethylene maintains its mechanical properties but is less effective at removing free
radicals, leaving the polyethylene more susceptible to oxidation. However, both annealed and remelted
polyethylene have shown in vivo oxidation.

RECOMMENDED READINGS
Currier BH, Van Citters DW, Currier JH, Collier JP. In vivo oxidation in remelted highly cross-linked
retrievals. J Bone Joint Surg Am. 2010 Oct 20;92(14):2409-18. PubMed PMID: 20962191.
Kurtz SM, Austin MS, Azzam K, Sharkey PF, MacDonald DW, Medel FJ, Hozack WJ. Mechanical
properties, oxidation, and clinical performance of retrieved highly cross-linked Crossfire liners after
intermediate-term implantation. J Arthroplasty. 2010 Jun;25(4):614-23.e1-2. Epub 2009 Jun 10. PubMed
PMID: 19520545.
Kuzyk PR, Saccone M, Sprague S, Simunovic N, Bhandari M, Schemitsch EH. Cross-linked versus
conventional polyethylene for total hip replacement: a meta-analysis of randomised controlled trials. J
Bone Joint Surg Br. 2011 May;93(5):593-600. Review. PubMed PMID: 21511923.
McKellop H, Shen FW, Lu B, Campbell P, Salovey R. Effect of sterilization method and other
modifications on the wear resistance of acetabular cups made of ultra-high molecular weight polyethylene.
A hip-simulator study. J Bone Joint Surg Am. 2000 Dec;82-A(12):1708-25. PubMed PMID: 11130644.

© 2013 American Academy of Orthopaedic Surgeons 2013 Adult Reconstructive Surgery of the Hip and Knee Self-Assessment Examination
2013 Adult Reconstructive Surgery of the Hip and Knee Self-Assessment Examination Answer Book • 19

Question 9
A 68-year-old woman had advanced right knee arthritis and total knee replacement was planned. She
learned she had primary biliary cirrhosis at age 41 and now has advancing liver failure. Preoperative
coagulation tests show a baseline International Normalized Ratio (INR) of 1.36. Appropriate methods
to prevent thromboembolic disease as recommended by the 2011 AAOS Clinical Practice Guideline,
Preventing Venous Thromboembolic Disease in Patients Undergoing Elective Hip and Knee Arthroplasty,
include

1. use of mechanical prophylaxis (eg, pneumatic calf compressors) while in the hospital.
2. oral warfarin with a goal INR between 2.0 and 3.0.
3. low-dose warfarin for 3 weeks postsurgically beginning 48 hours after surgery.
4. no prophylaxis because this patient already is partially anticoagulated secondary to her liver
disease.

PREFERRED RESPONSE: 1

DISCUSSION
The 2011 AAOS Clinical Practice Guideline, Preventing Venous Thromboembolic Disease in Patients
Undergoing Elective Hip and Knee Arthroplasty, recommends the use of mechanical prophylaxis
for patients at increased risk for bleeding (including those with liver disease or hemophilia). This
recommendation is the consensus of the workgroup that established these guidelines because there was
insufficient evidence to justify a stronger recommendation in this clinical scenario. The other responses
use no prophylaxis or pharmacological prophylaxis. Pharmacological prophylaxis is not recommended in
patients who are at increased risk for bleeding.

RECOMMENDED READINGS
American Academy of Orthopaedic Surgeons: Preventing Venous Thromboembolic Disease in Patients
Undergoing Elective Hip and Knee Arthroplasty. Rosemont, IL: American Academy of Orthopaedic
Surgeons, September 2011. Available at http://www.aaos.org/research/guidelines/VTE/VTE_guideline.asp
Accessed 8/21/2012.

© 2013 American Academy of Orthopaedic Surgeons 2013 Adult Reconstructive Surgery of the Hip and Knee Self-Assessment Examination
20 • American Academy of Orthopaedic Surgeons

Figure 10a Figure 10b

Question 10
A healthy, active 72-year-old man tripped and fell, landing on his left hip 10 weeks after an uncomplicated
left primary uncemented total hip replacement. A radiograph taken 6 weeks after surgery and before the
fall is shown in Figure 10a. A radiograph taken after the fall is shown in Figure 10b. He was unable to
bear weight and was brought to the emergency department. Examination revealed a slightly shortened left
lower extremity and some mild ecchymosis just distal to the left greater trochanteric region, but his skin
was intact without abrasions or lacerations. What is the most appropriate treatment?

1. Open reduction and cerclage fixation of the fracture


2. Open reduction and revision of the femoral implant to a long cemented stem
3. Open reduction and revision of the femoral implant to a long fluted and tapered uncemented
stem
4. Application of balanced traction and surgery after the ecchymosis has resolved

PREFERRED RESPONSE: 3

DISCUSSION
This patient has a periprosthetic femoral fracture with a loose femoral stem and normal femoral bone stock
(Vancouver type B2). The most appropriate treatment is fixation of the fracture along with revision of the
stem. Considering his age, bone quality, and activity level, a longer uncemented stem is most predictable.
Although a cylindrical stem may also be used, the fluted stem option is the only uncemented choice listed
and the most appropriate response. A cemented stem is a poorer choice because it is difficult to keep the
cement out of the fracture site; this would pose risk for nonunion at the fracture, and overall poorer results
have been associated with long cemented stems in healthy, active people. Surgery does not need to be
delayed to allow the ecchymosis to resolve, and simple open reduction and fixation does not address the
loose stem.

© 2013 American Academy of Orthopaedic Surgeons 2013 Adult Reconstructive Surgery of the Hip and Knee Self-Assessment Examination
2013 Adult Reconstructive Surgery of the Hip and Knee Self-Assessment Examination Answer Book • 21

RECOMMENDED READINGS
Mulay S, Hassan T, Birtwistle S, Power R. Management of types B2 and B3 femoral periprosthetic
fractures by a tapered, fluted, and distally fixed stem. J Arthroplasty. 2005 Sep;20(6):751-6. PubMed
PMID: 16139712.
Ko PS, Lam JJ, Tio MK, Lee OB, Ip FK. Distal fixation with Wagner revision stem in treating Vancouver
type B2 periprosthetic femur fractures in geriatric patients. J Arthroplasty. 2003 Jun;18(4):446-52.
PubMed PMID: 12820087.
Berry DJ. Treatment of Vancouver B3 periprosthetic femur fractures with a fluted tapered stem. Clin
Orthop Relat Res. 2003 Dec;(417):224-31. PubMed PMID: 14646721.

Question 11
A 78-year-old active woman who weighs 227 pounds has a long history of type 2 diabetes mellitus and
had a total knee replacement 15 years ago. She underwent revision arthroplasty for loose components.
After surgery, she fell and now has a 35-degree extensor lag with a high-riding patella on the lateral
radiographic view. When attempting to surgically repair the torn extensor, gross purulence is found,
leading to a resection with an antibiotic cement spacer. Enterococcus bacteria are recovered on cultures a
few days later; this species is resistant to several antibiotics. The wound drains for 3 weeks and then heals
with continued redness, pain, and swelling despite intravenous antibiotics. What is the most effective
long-term treatment for this patient?

1. Knee fusion
2. Primary extensor repair with a hinged total knee
3. Revision total knee with extensor mechanism allograft
4. Repeat debridement and chronic antibiotic suppression

PREFERRED RESPONSE: 1

DISCUSSION
Arthrodesis may be the most satisfactory long-term option for this elderly, obese patient even though all
knee motion will be lost. The other options presented are associated with unpredictable results and higher
failure rates.

RECOMMENDED READINGS
Rand JA. Alternatives to reimplantation for salvage of the total knee arthroplasty complicated by infection.
J Bone Joint Surg Am. 1993 Feb;75(2):282-9. Review. PubMed PMID: 8423191.
MacDonald JH, Agarwal S, Lorei MP, Johanson NA, Freiberg AA. Knee arthrodesis. J Am Acad Orthop
Surg. 2006 Mar;14(3):154-63. PubMed PMID: 16520366.

© 2013 American Academy of Orthopaedic Surgeons 2013 Adult Reconstructive Surgery of the Hip and Knee Self-Assessment Examination
22 • American Academy of Orthopaedic Surgeons

CLINICAL SITUATION FOR QUESTIONS 12 AND 13


A 65-year-old woman with type 2 diabetes mellitus and hypertension who underwent an index total knee
arthroplasty (TKA) 1 year ago has a knee aspirate culture positive methicillin-resistant Staphylococcus
aureus periprosthetic joint infection after 2 days of increasing pain and swelling. She states her knee
“never felt right.” Her erythrocyte sedimentation rate (ESR) and C-reactive protein (CRP) levels are
elevated. Radiographs reveal well-fixed, appropriately positioned components.

Question 12
What is the most appropriate treatment?

1. Open debridement, poly exchange, and intravenous (IV) antibiotics


2. Two-stage exchange arthroplasty
3. Arthroscopic debridement
4. Resection arthroplasty

PREFERRED RESPONSE: 2

Question 13
The patient undergoes a successful first stage that includes removal of implants and placement of an
articulating spacer. IV antibiotics are administered for 6 weeks. Appropriate clinical management for this
patient includes

1. leukocyte-labeled imaging.
2. reimplantation surgery only after her ESR and CRP values are within defined limits.
3. obtaining an intraoperative culture at the time of 2-stage exchange.
4. obtaining intraoperative gram stain at the time of the 2-stage exchange.

PREFERRED RESPONSE: 3

DISCUSSION FOR QUESTIONS 12 AND 13


This patient has had a painful knee joint since surgery, and it is possible this is a chronic infection. In
this setting, a 2-stage reimplantation procedure is most appropriate. There is no role for arthroscopic
debridement and resection arthroplasty is not indicated. During the second stage, it is important to obtain
cultures to confirm successful eradication of the infection. If there is clinical suspicion for persistent
joint infection, an intraoperative frozen section can be indicated, but gram stain is useful. Aspiration
and culture prior to the intraoperative culture should be obtained after the patient has not been taking the
antibiotic for a minimum of 2 weeks. ESR and CRP trends are more valuable than the actual numbers in
this setting. Finally, leukocyte-labeled imaging has no role in this setting.

© 2013 American Academy of Orthopaedic Surgeons 2013 Adult Reconstructive Surgery of the Hip and Knee Self-Assessment Examination
2013 Adult Reconstructive Surgery of the Hip and Knee Self-Assessment Examination Answer Book • 23

RECOMMENDED READINGS FOR QUESTIONS 12 AND 13


Deirmengian C, Greenbaum J, Lotke PA, Booth RE Jr, Lonner JH. Limited success with open debridement
and retention of components in the treatment of acute Staphylococcus aureus infections after total knee
arthroplasty. J Arthroplasty. 2003 Oct;18(7 Suppl 1):22-6. PubMed PMID: 14560406.
Bradbury T, Fehring TK, Taunton M, Hanssen A, Azzam K, Parvizi J, Odum SM. The fate of acute
methicillin-resistant Staphylococcus aureus periprosthetic knee infections treated by open debridement
and retention of components. J Arthroplasty. 2009 Sep;24(6 Suppl):101-4. Epub 2009 Jun 24. Review.
PubMed PMID: 19553077.
Krempec JA, Masonis JL, Fehring TK. Infection in total knee arthroplasty. In: Glassman AH, Lachiewicz
PF, Tanzer M, eds. Orthopaedic Knowledge Update: Hip and Knee Reconstruction 4. Rosemont, IL:
American Academy of Orthopaedic Surgeons; 2011:201-215.
Della Valle C, Parvizi J, Bauer TW, DiCesare PE, Evans RP, Segreti J, Spangehl M, Watters WC 3rd,
Keith M, Turkelson CM, Wies JL, Sluka P, Hitchcock K; American Academy of Orthopaedic Surgeons.
American Academy of Orthopaedic Surgeons clinical practice guideline on: the diagnosis of periprosthetic
joint infections of the hip and knee. J Bone Joint Surg Am. 2011 Jul 20;93(14):1355-7. PubMed PMID:
21792503.

END OF SERIES

Question 14
A 77-year-old man who had right total knee replacement surgery 2½ years ago has had knee pain since
surgery. The pain is diffuse, constant, and made worse with activity. He notes warmth and swelling in
his knee. Examination shows a well-healed incision, no erythema, moderate warmth, synovitis, and an
effusion. The knee is stable, and has an arc of flexion between 3 and 120 degrees. Radiographs show
well-fixed and well-aligned implants. What is the most appropriate initial treatment?

1. Knee aspiration for culture


2. CT scan of the knee to assess implant rotation
3. Indium, technetium-sulfur colloid scan of the knee
4. Erythrocyte sedimentation rate (ESR) and C-reactive protein (CRP) labs

PREFERRED RESPONSE: 4

DISCUSSION
This patient’s history and physical findings are concerning for deep infection. Inflammatory markers
(ESR and CRP) should first be obtained, and, if levels are elevated, proceed to knee aspiration for synovial
cell count and culture. A bone scan is not indicated in an initial investigation for deep infection, rarely is
helpful, and is not cost effective. A CT scan to assess implant rotation is an appropriate investigation for
knee pain when the clinical scenario is not suspicious for a deep infection and when infection has been
excluded.

© 2013 American Academy of Orthopaedic Surgeons 2013 Adult Reconstructive Surgery of the Hip and Knee Self-Assessment Examination
24 • American Academy of Orthopaedic Surgeons

RECOMMENDED READINGS
Greidanus NV, Masri BA, Garbuz DS, Wilson SD, McAlinden MG, Xu M, Duncan CP. Use of
erythrocyte sedimentation rate and C-reactive protein level to diagnose infection before revision total knee
arthroplasty. A prospective evaluation. J Bone Joint Surg Am. 2007 Jul;89(7):1409-16. PubMed PMID:
17606776.
Della Valle CJ, Sporer SM, Jacobs JJ, Berger RA, Rosenberg AG, Paprosky WG. Preoperative testing for
sepsis before revision total knee arthroplasty. J Arthroplasty. 2007 Sep;22(6 Suppl 2):90-3. Epub 2007 Jul
26. PubMed PMID: 17823024.
American Academy of Orthopaedic Surgeons: Diagnosis of Periprosthetic Joint Infections of the Hip and
Knee. Rosemont, IL: American Academy of Orthopaedic Surgeons, June 2010. Available at http://www.
aaos.org/research/guidelines/PJIguideline.asp Accessed 8/21/2012.

Figure 15a Figure 15b

Question 15
Figures 15a and 15b are the 6-week postsurgical anteroposterior hip radiograph and current radiograph
of a 54-year-old avid hiker who returns for routine follow-up 3 years after an uncomplicated uncemented
modular metal-on-metal hip replacement. He reports mild activity-related aching diffusely around the
right hip region, but does not feel restricted with his activities. Examination reveals no local tenderness, a
well-healed incision, and mild discomfort at the extremes of rotation. An erythrocyte sedimentation rate
(ESR) and C-reactive protein (CRP) are obtained, showing 9 mm/h (reference range, 0-20 mm/h) and 2.0
mg/L (reference range, 0.08-3.1 mg/L), respectively. What is the etiology of the radiographic finding?

1. Osteolysis secondary to metal particle wear (an adverse reaction to metal debris)
2. Osteolysis secondary to loosening of the femoral implant
3. Metastatic lesion to the proximal femur
4. Deep periprosthetic joint infection

PREFERRED RESPONSE: 1

© 2013 American Academy of Orthopaedic Surgeons 2013 Adult Reconstructive Surgery of the Hip and Knee Self-Assessment Examination
2013 Adult Reconstructive Surgery of the Hip and Knee Self-Assessment Examination Answer Book • 25

DISCUSSION
The radiograph shows a large area of osteolysis involving the proximal femur. The implants appear
solidly fixed on radiographs and, in the absence of symptoms, it is unlikely the implants are loose or
infected. Although infection or metastatic disease is a possibility, the normal ESR and CRP values make
the diagnosis of deep infection unlikely. The most likely cause for the radiographic findings is wear from
metal particles resulting in an adverse local tissue response and osteolysis.

RECOMMENDED READINGS
Williams DH, Greidanus NV, Masri BA, Duncan CP, Garbuz DS. Prevalence of Pseudotumor in
Asymptomatic Patients After Metal-on-Metal Hip Arthroplasty. J Bone Joint Surg Am. 2011 Dec
7;93(23):2164-71. PubMed PMID: 22159851.
Kwon YM, Ostlere SJ, McLardy-Smith P, Athanasou NA, Gill HS, Murray DW. “Asymptomatic”
pseudotumors after metal-on-metal hip resurfacing arthroplasty: prevalence and metal ion study. J
Arthroplasty. 2011 Jun;26(4):511-8. Epub 2010 Jun 29. PubMed PMID: 20591612.
Hart AJ, Ilo K, Underwood R, Cann P, Henckel J, Lewis A, Cobb J, Skinner J. The relationship between
the angle of version and rate of wear of retrieved metal-on-metal resurfacings: a prospective, CT-based
study. J Bone Joint Surg Br. 2011 Mar;93(3):315-20. Review. PubMed PMID: 21357951.

Question 16
What risk factor is associated with a poor prognosis after revision of a failed metal-on-metal resurfacing
hip arthroplasty to total hip arthroplasty?

1. Femoral neck fracture


2. Osteonecrosis of the femoral head
3. Aseptic loosening of the femoral component
4. Pseudotumor formation

PREFERRED RESPONSE: 4

DISCUSSION
Recent clinical series have focused on the high complication rates and problems resulting from revision
cases with pseudotumor formation. Pseudotumors are probably related to very high levels of cobalt and
chrome particulate wear debris. The underlying etiology most likely is edge wear or impingement that
destroys the congruity of the articulation. Revision surgery was quite difficult in one series, particularly
if there were large amounts of soft-tissue destruction or nerve involvement. There was a high re-revision
rate, often resulting from recurrent pseudotumors with large amounts of debris that could not be removed
with the initial revision. The other causes of revision that can cause failure of the femoral resurfacing
component can be easily revised with a femoral stem component, and results can be nearly as favorable as
with a primary hip replacement.

© 2013 American Academy of Orthopaedic Surgeons 2013 Adult Reconstructive Surgery of the Hip and Knee Self-Assessment Examination
26 • American Academy of Orthopaedic Surgeons

RECOMMENDED READINGS
Grammatopolous G, Pandit H, Kwon YM, Gundle R, McLardy-Smith P, Beard DJ, Murray DW, Gill HS.
Hip resurfacings revised for inflammatory pseudotumour have a poor outcome. J Bone Joint Surg Br. 2009
Aug;91(8):1019-24. PubMed PMID: 19651827.
Pandit H, Glyn-Jones S, McLardy-Smith P, Gundle R, Whitwell D, Gibbons CL, Ostlere S, Athanasou N,
Gill HS, Murray DW. Pseudotumours associated with metal-on-metal hip resurfacings. J Bone Joint Surg
Br. 2008 Jul;90(7):847-51. PubMed PMID: 18591590.

Question 17
Ten-year follow-up studies of total hip replacements performed with modern alumina ceramic femoral
heads and acetabular liners show what outcomes?

1. Low incidence of osteolysis, squeaking noise, and ceramic head fractures


2. Same incidence of osteolysis as metal-polyethylene total hips of the same design
3. Higher incidence of osteolysis in hips that make audible noises in vivo
4. Elimination of ceramic head fractures resulting from use of improved biomaterials

PREFERRED RESPONSE: 1

DISCUSSION
Ten-year follow-up data have been published from a number of clinical centers worldwide that describe
the outcomes of total hip arthroplasties performed with third-generation alumina ceramic bearings (metal-
polyethylene control hips often were included in the same series). These studies show no osteolysis
around well-fixed metal components, and a small incidence of revision surgery to address bearing noise
(squeaking) or ceramic femoral head fracture. Newer ceramic materials are associated with satisfactory
outcomes in terms of elimination of wear-mediated osteolysis, but problems such as bearing noise and
catastrophic femoral head failure have not been eliminated even though the risk for these complications is
small and much improved compared to risk associated with earlier generations of ceramic bearings.

RECOMMENDED READINGS
Kim YH, Choi Y, Kim JS. Cementless total hip arthroplasty with ceramic-on-ceramic bearing in patients
younger than 45 years with femoral-head osteonecrosis. Int Orthop. 2010 Dec;34(8):1123-7. Epub 2009
Sep 26. PubMed PMID: 19784647.
D’Antonio JA, Sutton K. Ceramic materials as bearing surfaces for total hip arthroplasty. J Am Acad
Orthop Surg. 2009 Feb;17(2):63-8. PubMed PMID: 19202119.

© 2013 American Academy of Orthopaedic Surgeons 2013 Adult Reconstructive Surgery of the Hip and Knee Self-Assessment Examination
2013 Adult Reconstructive Surgery of the Hip and Knee Self-Assessment Examination Answer Book • 27

Question 18
When do most symptomatic thromboembolic events occur after undergoing total joint arthroplasty?

1. On the day of surgery


2. Within the first week after surgery
3. Between 1 week and 6 weeks after surgery
4. More than 3 months after surgery

PREFERRED RESPONSE: 3

DISCUSSION
Most clinical venous thromboembolism events occur between the second and sixth weeks after surgery. It
is estimated that 10% of patients are readmitted to the hospital within the first 3 months after total hip or
knee arthroplasties. Most pulmonary events on the day of surgery are related to fat embolism or cardiac
events.

RECOMMENDED READINGS
Oster G, Ollendorf DA, Vera-Llonch M, Hagiwara M, Berger A, Edelsberg J. Economic consequences of
venous thromboembolism following major orthopedic surgery. Ann Pharmacother. 2004 Mar;38(3):377-
82. Epub 2004 Jan 12. PubMed PMID: 14742829.
Friedman RJ, Gallus AS, Cushner FD, Fitzgerald G, Anderson FA Jr; Global Orthopaedic Registry
Investigators. Physician compliance with guidelines for deep-vein thrombosis prevention in total hip and
knee arthroplasty. Curr Med Res Opin. 2008 Jan;24(1):87-97. PubMed PMID: 18028586.

Question 19
What effect does morbid obesity (body mass index [BMI] higher than 40) have on total knee arthroplasty
outcomes?

1. No difference in functional outcome


2. Complication rates are similar to those experienced by nonobese patients
3. Revision rates are similar to those experienced by nonobese patients
4. More postoperative radiolucent lines

PREFERRED RESPONSE: 4

DISCUSSION
Patients with a BMI higher than 40 are more likely than nonobese patients to have radiolucent lines on
postoperative radiographs. These patients have a higher rate of complications, inferior survivorship with a
higher rate of revision, and poorer function scores.

© 2013 American Academy of Orthopaedic Surgeons 2013 Adult Reconstructive Surgery of the Hip and Knee Self-Assessment Examination
28 • American Academy of Orthopaedic Surgeons

RECOMMENDED READINGS
Amin AK, Clayton RA, Patton JT, Gaston M, Cook RE, Brenkel IJ. Total knee replacement in morbidly
obese patients. Results of a prospective, matched study. J Bone Joint Surg Br. 2006 Oct;88(10):1321-6.
PubMed PMID: 17012421.
Malinzak RA, Ritter MA, Berend ME, Meding JB, Olberding EM, Davis KE. Morbidly obese, diabetic,
younger, and unilateral joint arthroplasty patients have elevated total joint arthroplasty infection rates. J
Arthroplasty. 2009 Sep;24(6 Suppl):84-8. Epub 2009 Jul 15. PubMed PMID: 19604665.

Question 20
A 70-year-old woman who underwent total knee replacement 18 months ago has had 3 weeks of moderate
drainage from a previously healed wound. What is the most appropriate treatment?

1. Wound vacuum-assisted closure dressing


2. IV antibiotics for 6 weeks flowed by long term oral antibiotics
3. Irrigation and debridement and polyethylene exchange
4. Two-stage debridement and reconstruction

PREFERRED RESPONSE: 4

DISCUSSION
This situation represents a definitively and chronically infected knee replacement. Antibiotic therapy
alone may suppress the infection but will not eradicate it. Debridement and polyethylene exchange would
be appropriate treatment for an early postoperative infection. The treatment of choice is to perform a
2-stage debridement and reconstruction. Although not among the listed choices, an aspiration or culture
can be done presurgically and may help clinicians identify the best antibiotics to treat the condition.
Antibiotic selection would not affect the need for the 2-stage reconstruction, however.

RECOMMENDED READINGS
Mittal Y, Fehring TK, Hanssen A, Marculescu C, Odum SM, Osmon D. Two-stage reimplantation for
periprosthetic knee infection involving resistant organisms. J Bone Joint Surg Am. 2007 Jun;89(6):1227-
31. PubMed PMID: 17545425.
Schinsky MF, Della Valle CJ, Sporer SM, Paprosky WG. Perioperative testing for joint infection in
patients undergoing revision total hip arthroplasty. J Bone Joint Surg Am. 2008 Sep;90(9):1869-75.
Erratum in: J Bone Joint Surg Am. 2010 Mar;92(3):707. PubMed PMID: 18762646.

© 2013 American Academy of Orthopaedic Surgeons 2013 Adult Reconstructive Surgery of the Hip and Knee Self-Assessment Examination
2013 Adult Reconstructive Surgery of the Hip and Knee Self-Assessment Examination Answer Book • 29

Question 21
A 66-year-old woman had fever, chills, and increasing pain in her right hip. She underwent a total hip
arthroplasty using large-head metal-on-metal articulation 4 years earlier without complications. Her hip
pain began about 1 month ago following several days of productive cough that her primary care physician
had diagnosed as a viral illness. She had elevated serology with an erythrocyte sedimentation rate of 70
mm/h (reference range, 0-20 mm/h) and C-reactive protein of 3.5 mg/L (reference range, 0.08-3.1 mg/L).
There is no radiographic evidence of loosening or adverse bone remodeling around the hip arthroplasty.
What is the most appropriate course of action?

1. Hip aspiration
2. Metal artifact reduction sequence (MARS) MRI
3. Initiate intravenous antibiotics
4. Assess serum metal trace element levels

PREFERRED RESPONSE: 1

DISCUSSION
This patient has a history and laboratory values highly suggestive of infection following her upper-
respiratory-tract infection. Hip aspiration is the most appropriate work-up in this acute setting. Metal
ion level assessment is not useful in the workup for acute infection. A MARS MRI scan would be less
effective in diagnosing infection than hip aspiration. Empirical use of antibiotics is not appropriate.

RECOMMENDED READINGS
Della Valle C, Parvizi J, Bauer TW, DiCesare PE, Evans RP, Segreti J, Spangehl M, Watters WC 3rd,
Keith M, Turkelson CM, Wies JL, Sluka P, Hitchcock K; American Academy of Orthopaedic Surgeons.
American Academy of Orthopaedic Surgeons clinical practice guideline on: the diagnosis of periprosthetic
joint infections of the hip and knee. J Bone Joint Surg Am. 2011 Jul 20;93(14):1355-7. PubMed PMID:
21792503.
Parvizi J, Ghanem E, Menashe S, Barrack RL, Bauer TW. Periprosthetic infection: what are the diagnostic
challenges? J Bone Joint Surg Am. 2006 Dec;88 Suppl 4:138-47. PubMed PMID: 17142443.

Question 22
When comparing arthroscopic lavage and knee debridement to placebo in patients with chronic
symptomatic osteoarthritis, what outcome has been demonstrated?

1. Reliable and durable pain relief


2. No significant benefit for chronic osteoarthritis
3. Up to 75% pain relief for 2 months, then variable response
4. Three-month measurable pain relief, followed by recurrence

PREFERRED RESPONSE: 2

© 2013 American Academy of Orthopaedic Surgeons 2013 Adult Reconstructive Surgery of the Hip and Knee Self-Assessment Examination
30 • American Academy of Orthopaedic Surgeons

DISCUSSION
Excluding a diagnosis of meniscal tear, loose body, or mechanical derangement, when knee osteoarthritis
of indeterminate cause is treated with arthroscopic lavage and debridement, no discernable benefit has
been found to offset the risk of surgery. Effects of arthroscopy have not been clinically significant in the
vast majority of patient-oriented outcomes measures for pain and function at multiple time points between
1 week and 2 years after surgery.

RECOMMENDED READINGS
Moseley JB, O’Malley K, Petersen NJ, Menke TJ, Brody BA, Kuykendall DH, Hollingsworth JC, Ashton
CM, Wray NP. A controlled trial of arthroscopic surgery for osteoarthritis of the knee. N Engl J Med. 2002
Jul 11;347(2):81-8. PubMed PMID: 12110735.
Laupattarakasem W, Laopaiboon M, Laupattarakasem P, Sumananont C. Arthroscopic debridement for
knee osteoarthritis. Cochrane Database Syst Rev. 2008 Jan 23;(1):CD005118. Review. PubMed PMID:
18254069.

CLINICAL SITUATION FOR QUESTIONS 23 AND 24


A 57-year-old woman reported pain 1 year after total knee arthroplasty (TKA). The pain was
characterized as a sharp catching anterior pain that was aggravated by rising from a chair or climbing
stairs. Physical examination revealed a mild effusion and a range of motion of 2 to 130 degrees with
patellar crepitus. Symptoms were reproduced by resisted knee extension. Radiographs showed a well-
aligned posterior-stabilized TKA without evidence of component loosening.

Question 23
What is the most likely cause of this patient’s pain?

1. Patella clunk syndrome


2. Flexion gap instability
3. Polyethylene wear
4. Femoral component malrotation

PREFERRED RESPONSE: 1

© 2013 American Academy of Orthopaedic Surgeons 2013 Adult Reconstructive Surgery of the Hip and Knee Self-Assessment Examination
2013 Adult Reconstructive Surgery of the Hip and Knee Self-Assessment Examination Answer Book • 31

Question 24
What is the recommended treatment for this patient?

1. Physical therapy
2. Arthroscopic synovectomy
3. Tibial insert revision
4. Femoral component revision

PREFERRED RESPONSE: 2

DISCUSSION FOR QUESTIONS 23 AND 24


Patella clunk syndrome is caused by the development of a fibrous nodule on the posterior aspect of the
quadriceps tendon at its insertion into the patella. It causes a painful catching sensation when the extensor
mechanism traverses over the trochlear notch as the knee extends from 45 degrees of flexion to 30 degrees
from full extension. It characteristically occurs in posterior stabilized total knees and appears to be
related to femoral component design. The syndrome can usually be prevented by excising the residual
synovial fold just proximal to the patella. Flexion gap instability can also be a cause of a painful total
knee arthroplasty, but is less common in posterior stabilized implants. Femoral component malrotation
can be a cause of pain attributable to flexion gap imbalance or patella tracking problems. Polyethylene
wear would be unlikely after just 1 year. Patella clunk syndrome can usually be successfully addressed
with arthroscopic synovectomy and recurrence is uncommon. Physical therapy may help to strengthen the
quadriceps following synovectomy, but will not resolve the clunk syndrome symptoms. Femoral or tibial
insert revision is not indicated if patella clunk syndrome is the only problem resulting in a painful total
knee arthroplasty.

RECOMMENDED READINGS FOR QUESTIONS 23 AND 24


Dennis DA. Evaluation of painful total knee arthroplasty. J Arthroplasty. 2004 Jun;19(4 Suppl 1):35-40.
Review. PubMed PMID: 15190547.
Dajani KA, Stuart MJ, Dahm DL, Levy BA. Arthroscopic treatment of patellar clunk and synovial
hyperplasia after total knee arthroplasty. J Arthroplasty. 2010 Jan;25(1):97-103. Epub 2008 Dec 23.
PubMed PMID: 19106026.

END OF SERIES

© 2013 American Academy of Orthopaedic Surgeons 2013 Adult Reconstructive Surgery of the Hip and Knee Self-Assessment Examination
32 • American Academy of Orthopaedic Surgeons

Figure 25

Question 25
Figure 25 is the anteroposterior hip radiograph of a 74-year-old healthy and active man who was seen in
the office 18 months after a primary uncemented total hip replacement with a history of 3 hip dislocations.
The last dislocation occurred 1 week ago and he was treated in the emergency department with a closed
reduction and application of a hip abduction brace. All episodes of dislocation occurred when bending
forward. Aside from the episodes of dislocation, his hip functions well. Examination revealed a normal
gait with good abductor strength and pain-free hip movement. What is the most appropriate next
treatment step?

1. Prescribe physical therapy to work on abductor strengthening and reinforce hip position
precautions.
2. Recommend revision of the acetabular component to change cup position and increase the
head and liner size.
3. Recommend revision of the head and liner to a larger size using an elevated or oblique liner.
4. Continue use of the hip abduction brace for 6 weeks and follow with physical therapy.

PREFERRED RESPONSE: 2

DISCUSSION
This patient has had 3 hip dislocations since his hip replacement, and the radiograph shows an
underanteverted cup. He will likely continue to dislocate and surgery is indicated. The anteroposterior
radiograph indicates that the cup is vertically oriented and not anteverted. His acetabular component is
malpositioned and should be revised to provide the highest likelihood for success. A simple head and
liner exchange with a malpositioned implant is less likely to succeed. Additionally, the radiograph shows
sufficient acetabular bone stock medial to the cup. The addition of acetabular revision caries a low risk
for increased morbidity and will allow a large cup with a larger femoral head with sufficient polyethylene
thickness.

© 2013 American Academy of Orthopaedic Surgeons 2013 Adult Reconstructive Surgery of the Hip and Knee Self-Assessment Examination
2013 Adult Reconstructive Surgery of the Hip and Knee Self-Assessment Examination Answer Book • 33

RECOMMENDED READINGS
Patel PD, Potts A, Froimson MI. The dislocating hip arthroplasty: prevention and treatment. J
Arthroplasty. 2007 Jun;22(4 Suppl 1):86-90. Review. PubMed PMID: 17570285.
Carter AH, Sheehan EC, Mortazavi SM, Purtill JJ, Sharkey PF, Parvizi J. Revision for recurrent instability:
what are the predictors of failure? J Arthroplasty. 2011 Sep;26(6 Suppl):46-52. Epub 2011 May 8. PubMed
PMID: 21550768.

Question 26
A 67-year-old man who underwent an uncomplicated hip arthroplasty 9 years ago has had a 1-week
history of groin pain with movement. Radiographs reveal a well-positioned, well-fixed cementless
arthroplasty with mild eccentricity of the femoral head within the polyethylene. His serum C-reactive
protein (CRP) level is 3.0 mg/L (reference range, 0.08-3.1 mg/L) and erythrocyte sedimentation rate
(ESR) is 5 mm/h (reference range, 0-20 mm/h). What is the most appropriate next step in management of
the patient?

1. Aspiration of the hip to rule out an infectious process


2. Complete blood count with differential
3. Observation
4. Bone scan

PREFERRED RESPONSE: 3

DISCUSSION
Both CRP and ESR results are negative, so aspiration of the hip to rule out periprosthetic hip infection is
not recommended. The pain may be the result of a noninfectious process such as polyethylene wear with
lysis or a muscle strain. A bone scan is of limited value, as is any further bloodwork. If the symptoms
continue, further imaging may be of value.

RECOMMENDED READINGS
Della Valle C, Parvizi J, Bauer TW, DiCesare PE, Evans RP, Segreti J, Spangehl M, Watters WC 3rd,
Keith M, Turkelson CM, Wies JL, Sluka P, Hitchcock K; American Academy of Orthopaedic Surgeons.
American Academy of Orthopaedic Surgeons clinical practice guideline on: the diagnosis of periprosthetic
joint infections of the hip and knee. J Bone Joint Surg Am. 2011 Jul 20;93(14):1355-7. PubMed PMID:
21792503.
Krempec JA, Masonis JL, Fehring TK. Infection in total knee arthroplasty. In: Glassman AH, Lachiewicz
PF, Tanzer M, eds. Orthopaedic Knowledge Update: Hip and Knee Reconstruction 4. Rosemont, IL:
American Academy of Orthopaedic Surgeons; 2011:201-215.

© 2013 American Academy of Orthopaedic Surgeons 2013 Adult Reconstructive Surgery of the Hip and Knee Self-Assessment Examination
34 • American Academy of Orthopaedic Surgeons

Question 27
Cryotherapy has been demonstrated to achieve what effect after total knee replacement?

1. Decreased transfusion requirement


2. Improved pain, swelling, and analgesia
3. Improved range of motion at the time of discharge
4. Better long-term knee range of motion

PREFERRED RESPONSE: 3

DISCUSSION
In a meta-analysis of randomized controlled trials on the efficacy of cryotherapy after total knee
arthroplasty, patients treated with cryotherapy had less blood loss but no difference in transfusion
requirements. There was better range of motion at the time of discharge from the hospital. There was no
improvement in pain, swelling, or analgesia requirements. Patients treated with cryotherapy did not have
better long-term range of motion.

RECOMMENDED READINGS
Adie S, Naylor JM, Harris IA. Cryotherapy after total knee arthroplasty a systematic review and meta-
analysis of randomized controlled trials. J Arthroplasty. 2010 Aug;25(5):709-15. Epub 2009 Sep 2.
Review. PubMed PMID: 19729279.
Husted H, Lunn TH, Troelsen A, Gaarn-Larsen L, Kristensen BB, Kehlet H. Why still in hospital after
fast-track hip and knee arthroplasty? Acta Orthop. 2011 Dec;82(6):679-84. Epub 2011 Nov 9. PubMed
PMID: 22066560.

Question 28
Compared to retention of the native patella in primary total knee arthroplasty, routine patella resurfacing is
associated with

1. no patellar complications.
2. an increased occurrence of anterior knee pain.
3. a decreased patellar fracture rate.
4. a decreased risk for revision surgery.

PREFERRED RESPONSE: 4

DISCUSSION
Despite concerns regarding fracture, osteonecrosis, and patellar clunk, the routine retention of the native
patella during primary total knee replacement is associated with a 20% to 30% increased revision risk in
large joint registries. In addition, the retention of the native patella results in a 5.7 revision surgery rate in
patients with anterior knee pain.

© 2013 American Academy of Orthopaedic Surgeons 2013 Adult Reconstructive Surgery of the Hip and Knee Self-Assessment Examination
2013 Adult Reconstructive Surgery of the Hip and Knee Self-Assessment Examination Answer Book • 35

RECOMMENDED READINGS
Lidgren L, Robertsson O. Annual Report 2008: The Swedish Knee Arthroplasty Register. Dept. of
Orthopedics, Lund University Hospital. Lund, Sweden: Wallin and Dalholm AB; 2008. http://www.knee.
nko.se/english/online/uploadedFiles/112_SVK_2008Engl_1.1.pdf (Accessed 8/21/2012).
Graves S, Davidson D, de Steiger R, Tomkins A. Australian Orthopaedic Association - National Joint
Replacement Registry 2008 Annual Report. Adelaide, Australia: Australian Orthopaedic Association;
2008:152. http://www.surfacehippy.info/aoanationalreg08.php (Accessed 8/21/2012).

Question 29
What clinical outcome is associated with total hip replacements that have metal-metal bearings (compared
to total hip replacements with metal-polyethylene bearings)?

1. Soft-tissue sarcomas
2. Similar revision rates at 5 years
3. Increased nephrotoxicity
4. Pseudotumors

PREFERRED RESPONSE: 4

DISCUSSION
Patients with metal-metal total hip bearings have higher levels of cobalt and chromium in the bloodstream,
but systemic migration of wear debris from total hip bearings is also common to total hip arthroplasties
with polyethylene bearings. There is no direct evidence that patients with metal-metal total hip
arthroplasties experience a higher incidence of cancer. Chromosome abnormalities have been detected
in patients with metal-metal hip bearings, and the clinical consequences of this finding remain unknown.
Also, pseudotumors can form around the periprosthetic joint space in response to localized metal ion
debris and the host inflammatory response, although these tumors are not specific for failed metal-
metal total hip arthroplasties. Metal-on-metal hip replacements have higher revision rates compared
to conventional hip replacements in multiple registry studies. Although metal-on-metal articulations
have not been shown to cause renal failure, they are not recommended in patients with chronic renal
insufficiency.

RECOMMENDED READINGS
Dunstan E, Ladon D, Whittingham-Jones P, Carrington R, Briggs TW. Chromosomal aberrations
in the peripheral blood of patients with metal-on-metal hip bearings. J Bone Joint Surg Am. 2008
Mar;90(3):517-22. PubMed PMID: 18310701.
Fujishiro T, Moojen DJ, Kobayashi N, Dhert WJ, Bauer TW. Perivascular and diffuse lymphocytic
inflammation are not specific for failed metal-on-metal hip implants. Clin Orthop Relat Res. 2011
Apr;469(4):1127-33. Epub 2010 Oct 29. PubMed PMID: 21046298.

© 2013 American Academy of Orthopaedic Surgeons 2013 Adult Reconstructive Surgery of the Hip and Knee Self-Assessment Examination
36 • American Academy of Orthopaedic Surgeons

Question 30
A 55-year-old man with unilateral osteoarthritis of the hip underwent a total hip arthroplasty using
cementless fixation. The acetabular cup was 52 mm and the femoral head was 28 mm and made of cobalt-
chromium alloy. The bearing surface was made of annealed highly cross-linked polyethylene, with an
estimated thickness of 6.5 mm. What should the orthopaedic surgeon tell the patient regarding wear of the
bearing surface?

1. A highly cross-linked polyethylene bearing has superior wear characteristics compared to a


conventional polyethylene bearing.
2. A highly cross-linked polyethylene bearing has similar wear characteristics compared to a
conventional polyethylene bearing.
3. The incidence of osteolysis is expected to be higher with highly cross-linked polyethylene than
with conventional polyethylene.
4. The volumetric wear rate would be lower if a 36-mm femoral head were used.

PREFERRED RESPONSE: 1

DISCUSSION
In a prospective, randomized clinical trial of 100 patients undergoing cementless total hip arthroplasties,
the investigators compared highly crossed-linked polyethylene to conventional polyethylene. All of
the femoral heads were 28 mm. The mean follow-up was 6.8 years. The mean head penetration was
0.003 mm/year for the highly cross-linked polyethylene group in comparison to 0.051 mm/year for the
conventional polyethylene group (P = .006). The improved wear is seen with larger-diameter heads as
well. The volumetric wear rate of highly cross-linked polyethylene is equivalent to slightly higher with
a larger head than a 28-mm head. Incidence of periarticular osteolysis is lower with highly cross-linked
polyethylene.

RECOMMENDED READINGS
McCalden RW, MacDonald SJ, Rorabeck CH, Bourne RB, Chess DG, Charron KD. Wear rate of highly
cross-linked polyethylene in total hip arthroplasty. A randomized controlled trial. J Bone Joint Surg Am.
2009 Apr;91(4):773-82. PubMed PMID: 19339560.
Huo MH, Stockton KG, Mont MA, Parvizi J. What’s new in total hip arthroplasty. J Bone Joint Surg Am.
2010 Dec 15;92(18):2959-72. Review. PubMed PMID: 21159996.

© 2013 American Academy of Orthopaedic Surgeons 2013 Adult Reconstructive Surgery of the Hip and Knee Self-Assessment Examination
2013 Adult Reconstructive Surgery of the Hip and Knee Self-Assessment Examination Answer Book • 37

Question 31
A 49-year-old active man has groin pain 3 years after undergoing an uneventful total hip replacement
using a cobalt-chrome femoral head articulating against a cobalt-chrome acetabular insert. The pain
intensifies with activity and travels down his thigh. Examination and radiographic evaluation are not
particularly helpful; there is no evidence of spinal or vascular disease. What is the next step in the
evaluation of this patient?

1. A 3-phase bone scan


2. Measurement of synovial metal ions levels
3. Erythrocyte sedimentation rate (ESR), C-reactive protein (CRP), and possible hip aspiration
4. Bearing exchange to a metal-polyethylene combination

PREFERRED RESPONSE: 3

DISCUSSION
ESR, CRP, and possible hip aspiration is the most logical next step even though at some point, bearing
exchange may emerge as the ultimate treatment for a metal-metal adverse reaction in this patient. But the
initial workup of a patient with a painful total hip that was otherwise functioning well must include the
differential diagnosis of infection, which must be excluded with an appropriate laboratory workup, clinical
history, and hip aspiration. The latter study may also help to diagnose a reaction to the metal bearing;
cobalt and chromium levels in the aspirate can be investigated, and the color and quantity of the aspirate
can be examined along with the cell count. Serum levels of metal ions at this stage could be both helpful
and difficult to interpret.

RECOMMENDED READINGS
Wynn-Jones H, Macnair R, Wimhurst J, Chirodian N, Derbyshire B, Toms A, Cahir J. Silent soft tissue
pathology is common with a modern metal-on-metal hip arthroplasty. Acta Orthop. 2011 Jun;82(3):301-7.
Epub 2011 Apr 19. PubMed PMID: 21504335.
Engh CA Jr, Ho H, Engh CA. Metal-on-metal hip arthroplasty: does early clinical outcome justify the
chance of an adverse local tissue reaction? Clin Orthop Relat Res. 2010 Feb;468(2):406-12. Epub 2009
Sep 1. PubMed PMID: 19727991.

© 2013 American Academy of Orthopaedic Surgeons 2013 Adult Reconstructive Surgery of the Hip and Knee Self-Assessment Examination
38 • American Academy of Orthopaedic Surgeons

Figure 32a Figure 32b Figure 32c

Question 32
Figures 32a and 32b are the radiographs of a 25-year-old woman whose pain has progressed during the
last several years to pain with any activity and pain at night. What is the most appropriate treatment?

1. Proximal tibial osteotomy


2. Distal femoral osteotomy
3. Lateral unicompartmental arthroplasty
4. Total knee arthroplasty

PREFERRED RESPONSE: 2

DISCUSSION
This patient is a good candidate for a joint-preserving procedure. Her symptoms and radiographic findings
reveal valgus malalignment of the knee with narrowing of the lateral joint space. The alignment can be
corrected with a varus-producing distal osteotomy as shown in Figure 32c. Most patients do not proceed
to knee arthroplasty for at least 10 years after this procedure. Osteotomy is preferred over partial or total
knee arthroplasty because of the patient’s young age. Varus proximal tibial osteotomy would result in
joint line obliquity.

RECOMMENDED READINGS
Backstein D, Morag G, Hanna S, Safir O, Gross A. Long-term follow-up of distal femoral varus osteotomy
of the knee. J Arthroplasty. 2007 Jun;22(4 Suppl 1):2-6. PubMed PMID: 17570268.
Kosashvili Y, Safir O, Gross A, Morag G, Lakstein D, Backstein D. Distal femoral varus osteotomy for
lateral osteoarthritis of the knee: a minimum ten-year follow-up. Int Orthop. 2010 Feb;34(2):249-54. Epub
2009 May 26. PubMed PMID: 19468727.

© 2013 American Academy of Orthopaedic Surgeons 2013 Adult Reconstructive Surgery of the Hip and Knee Self-Assessment Examination
2013 Adult Reconstructive Surgery of the Hip and Knee Self-Assessment Examination Answer Book • 39

Question 33
What is the plasma half-life of warfarin?

1. 1 to 2 hours
2. 4 to 6 hours
3. 12 to 18 hours
4. 36 to 42 hours

PREFERRED RESPONSE: 4

DISCUSSION
Warfarin, which is dosed daily, can take 72 to 96 hours to reach therapeutic levels. It has a plasma half-
life of 36 to 42 hours. Low-molecular heparins have a plasma half-life of 4 to 5 hours, and fondaparinux
has a half-life of 17 to 21 hours. Warfarin will not affect the International Normalized Ratio (INR) until 2
to 3 days after it is given. Patients on chronic warfarin therapy should have treatment stopped 3 to 5 days
before elective surgery to allow the INR to normalize.

RECOMMENDED READINGS
Colwell CW Jr, Hardwick ME. Venous thromboembolic disease and prophylaxis in total joint arthroplasty.
In: Barrack RL, Booth RE Jr, Lonner JH, McCarthy JC, Mont MA, Rubash HE, eds. Orthopaedic
Knowledge Update: Hip and Knee Reconstruction 3. Rosemont, IL: American Academy of Orthopaedic
Surgeons; 2006:233-240.
Della Valle CJ. Coagulation and thromboembolism. In: Lieberman JR, ed. AAOS Comprehensive
Orthopaedic Review. Vol 1. Rosemont, IL: American Academy of Orthopaedic Surgeons; 2009:149-156.

Question 34
An orthopaedic surgeon noticed a displaced calcar fracture during stem insertion when performing total
hip arthroplasty using cementless fixation. What is the most appropriate course of action?

1. Intraoperative exploration to determine the extent of the fracture


2. Use of a longer stem without fixation of the calcar fracture
3. Complete insertion of the stem and measures to protect the patient against full weight bearing
for 4 weeks
4. Removal of the stem, internal fixation of the fracture, and definitive reconstruction at a later
stage after the fracture has healed

PREFERRED RESPONSE: 1

© 2013 American Academy of Orthopaedic Surgeons 2013 Adult Reconstructive Surgery of the Hip and Knee Self-Assessment Examination
40 • American Academy of Orthopaedic Surgeons

DISCUSSION
Calcar fractures can occur with both cemented and cementless stem fixation during surgery. The distal
extent of the fracture must be identified either by direct visualization or intraoperative radiograph prior
to fixation or implantation of the femoral component. The recommended treatment is to fix the calcar
fracture with cerclage wires/cables to restore the mechanical stability of the femoral metaphysis. The
same stem can be inserted successfully. The majority of these fractures unite without adverse stem
fixation problems.

RECOMMENDED READINGS
Masri BA, Meek RM, Duncan CP. Periprosthetic fractures evaluation and treatment. Clin Orthop Relat
Res. 2004 Mar;(420):80-95. Review. PubMed PMID: 15057082.
Mont MA, Maar DC, Krackow KA, Hungerford DS. Hoop-stress fractures of the proximal femur during
hip arthroplasty. Management and results in 19 cases. J Bone Joint Surg Br. 1992 Mar;74(2):257-60.
PubMed PMID: 1544964.

Question 35
A 48-year-old woman had an 8-month history of spontaneous onset of left medial knee pain. She was
otherwise healthy with an unremarkable past medical history. Prior to the onset of knee pain, she jogged,
played tennis, and golfed regularly. She wished to remain active. Examination showed a fit woman with
a BMI of 26, a stable left knee with full range of motion, and some mild medial joint line tenderness.
Radiograph results were normal. An MRI scan showed diffuse grade 3 and a focal area of grade 4
chondromalacia on the medial femoral condyle. The medial meniscus had a degenerative signal but no
tear. The remainder of the knee showed no additional pathology. What is the most appropriate initial
treatment?

1. Lateral heal wedge


2. Low-impact aerobic exercises
3. Glucosamine 1500 mg/day and chondroitin sulfate 800 mg/day
4. Arthroscopic debridement and microfracture of the focal area of grade 4 chondromalacia to
reduce risk for progression

PREFERRED RESPONSE: 2

DISCUSSION
This patient has early medial compartmental osteoarthritis of her knee. According to the 2008 AAOS
Clinical Practice Guideline, Treatment of Osteoarthritis of the Knee (Non-Arthroplasty), there is Level
1 evidence and an “A” recommendation for the use of low-impact aerobic exercises. The guideline also
has “A” recommendations with Level 1 evidence indicating that glucosamine and chondroitin should
not be prescribed and that arthroscopic debridement not be performed in the absence of symptoms of a
meniscal tear or loose body. Lateral heal wedge is not appropriate; the AAOS guideline provides a “B”
recommendation with Level 2 evidence indicating that a lateral heal wedge not be prescribed.

© 2013 American Academy of Orthopaedic Surgeons 2013 Adult Reconstructive Surgery of the Hip and Knee Self-Assessment Examination
2013 Adult Reconstructive Surgery of the Hip and Knee Self-Assessment Examination Answer Book • 41

RECOMMENDED READINGS
American Academy of Orthopaedic Surgeons: Treatment of Osteoarthritis of the Knee. Rosemont, IL:
American Academy of Orthopaedic Surgeons, December 2008. Available at: http://www.aaos.org/research/
guidelines/GuidelineOAKnee.asp Accessed 8/21/2012.
Richmond J, Hunter D, Irrgang J, Jones MH, Levy B, Marx R, Snyder-Mackler L, Watters WC 3rd,
Haralson RH 3rd, Turkelson CM, Wies JL, Boyer KM, Anderson S, St Andre J, Sluka P, McGowan R;
American Academy of Orthopaedic Surgeons. Treatment of osteoarthritis of the knee (nonarthroplasty). J
Am Acad Orthop Surg. 2009 Sep;17(9):591-600. PubMed PMID: 19726743.

Figure 36

RESPONSES FOR QUESTIONS 36 AND 37


1. Stop physical therapy and institute oral antibiotics.
2. Stop physical therapy and institute intravenous (IV) antibiotics.
3. Open irrigation and debridement, polyethylene spacer exchange, and IV antibiotics
4. Remove components and insert an antibiotic spacer.

For each question below, please select the most appropriate treatment from the list above.

Question 36
Figure 36 is the postoperative photograph of a patient who underwent a total knee arthroplasty 10 days
after surgery. Knee aspiration suggests a Streptococcus infection.

PREFERRED RESPONSE: 3

© 2013 American Academy of Orthopaedic Surgeons 2013 Adult Reconstructive Surgery of the Hip and Knee Self-Assessment Examination
42 • American Academy of Orthopaedic Surgeons

Question 37
Seven weeks after total knee replacement surgery, a patient has a painful swollen knee. Knee aspiration
reveals coagulase-negative Staphylococcus aureus.

PREFERRED RESPONSE: 4

DISCUSSION FOR QUESTIONS 36 AND 37


An acute postoperative infection during the first 2 to 4 weeks should be treated with a return to the
operating room for open irrigation and debridement of the wound. Polyethylene spacer exchange aides
in washing out the entire knee joint. IV antibiotics are also indicated in this situation. To address
persistent wound drainage, there is no role for oral or IV antibiotics alone. Removal of the arthroplasty
components is recommended for infections after the initial 2- to 4-week postoperative period. However,
several recent publications demonstrate a failure rate higher than 50% when the organism is a methicillin-
resistant Staphylococcus aureus. Six weeks after surgery, this scenario is no longer considered an acute
postoperative infection, and most authors recommend a 2-stage protocol with removal of components and
placement of an antibiotic-impregnated cement spacer and 4 to 6 weeks of IV antibiotics.

RECOMMENDED READINGS FOR QUESTIONS 36 AND 37


Krempec JA, Masonis JL, Fehring TK. Infection in total knee arthroplasty. In: Glassman AH, Lachiewicz
PF, Tanzer M, eds. Orthopedic Knowledge Update: Hip and Knee Reconstruction 4. Rosemont, IL:
American Academy of Orthopaedic Surgeons; 2011:201-215.
Bradbury T, Fehring TK, Taunton M, Hanssen A, Azzam K, Parvizi J, Odum SM. The fate of acute
methicillin-resistant Staphylococcus aureus periprosthetic knee infections treated by open debridement
and retention of components. J Arthroplasty. 2009 Sep;24(6 Suppl):101-4. Epub 2009 Jun 24. Review.
PubMed PMID: 19553077.
Sherrell JC, Fehring TK, Odum S, Hansen E, Zmistowski B, Dennos A, Kalore N; Periprosthetic Infection
Consortium. The Chitranjan Ranawat Award: fate of two-stage reimplantation after failed irrigation and
debridement for periprosthetic knee infection. Clin Orthop Relat Res. 2011 Jan;469(1):18-25. PubMed
PMID: 20582495.

END OF SERIES

© 2013 American Academy of Orthopaedic Surgeons 2013 Adult Reconstructive Surgery of the Hip and Knee Self-Assessment Examination
2013 Adult Reconstructive Surgery of the Hip and Knee Self-Assessment Examination Answer Book • 43

Question 38
What is the difference in outcome when comparing high tibial osteotomy (HTO) to total knee arthroplasty
(TKA)?

1. TKA has a longer recovery period than HTO.


2. HTO provides more complete pain relief than TKA.
3. HTO is more reliable in older patients than TKA.
4. HTO outcomes among thin, active, young patients who undergo this procedure approach
outcomes associated with TKA.

PREFERRED RESPONSE: 4

DISCUSSION
The ideal candidate for HTO is a thin, active person with a stable knee, unicompartmental knee symptoms,
and age younger than 60. TKA offers a shorter recovery period and more complete pain relief than HTO.
TKA is believed to be more reliable than HTO for patients older than age 60.

RECOMMENDED READINGS
Hanssen AD, Stuart MJ, Scott RD, Scuderi GR. Surgical options for the middle-aged patient with
osteoarthritis of the knee joint. Instr Course Lect. 2001;50:499-511. Review. PubMed PMID: 11372352.
Richmond J, Hunter D, Irrgang J, Jones MH, Levy B, Marx R, Snyder-Mackler L, Watters WC 3rd,
Haralson RH 3rd, Turkelson CM, Wies JL, Boyer KM, Anderson S, St Andre J, Sluka P, McGowan R;
American Academy of Orthopaedic Surgeons. Treatment of osteoarthritis of the knee (nonarthroplasty). J
Am Acad Orthop Surg. 2009 Sep;17(9):591-600. PubMed PMID: 19726743.

© 2013 American Academy of Orthopaedic Surgeons 2013 Adult Reconstructive Surgery of the Hip and Knee Self-Assessment Examination
44 • American Academy of Orthopaedic Surgeons

Figure 39

Question 39
Figure 39 is a radiograph of a 72-year-old man who underwent an open reduction and internal fixation of
a right femoral neck fracture. After 3 months he started to develop pain, and during the next 8 months he
complained of progressive pain and shortening of the hip. What is the most appropriate treatment?

1. Girdlestone
2. Total hip replacement
3. Hardware removal
4. Hardware removal with revision open reductions and internal fixation

PREFERRED RESPONSE: 2

DISCUSSION
Even though a relatively short amount of time has passed since the index surgery, this patient has
developed significant osteonecrosis that has caused collapse of the bony structures and the hardware
prominent. Total hip replacement gives the most efficient pain relief. Hardware removal with or without
re-reduction does not provide reliable pain relief. A girdlestone does not allow the patient to function.

RECOMMENDED READINGS
Damany DS, Parker MJ, Chojnowski A. Complications after intracapsular hip fractures in young adults.
A meta-analysis of 18 published studies involving 564 fractures. Injury. 2005 Jan;36(1):131-41. Review.
PubMed PMID: 15589931.
Mullis BH, Anglen J. Hip trauma. In: Flynn JM, ed. Orthopaedic Knowledge Update 10. Rosemont, IL:
American Academy of Orthopaedic Surgeons; 2011:399-411.

© 2013 American Academy of Orthopaedic Surgeons 2013 Adult Reconstructive Surgery of the Hip and Knee Self-Assessment Examination
2013 Adult Reconstructive Surgery of the Hip and Knee Self-Assessment Examination Answer Book • 45

Figure 40

Question 40
Figure 40 is the radiograph of a 68-year-old woman who has right knee pain that is limiting her activity
and severe preoperative valgus deformity. During total knee arthroplasty, what pathologic features are
typically encountered?

1. Lateral femoral hypoplasia


2. Internal rotation of the tibia relative to the femur
3. Medial patella tracking
4. Tight medial collateral ligament

PREFERRED RESPONSE: 1

DISCUSSION
In patients with severe valgus deformity, problems frequently encountered include loose or attenuated
medial collateral ligament, tight lateral retinaculum and lateral ligamentous structures (lateral collateral,
posterolateral corner), atrophic lateral femoral condyle, lateral patella tracking, and external rotation
of the tibia relative to the femur. The hypoplastic lateral condyle can cause internal rotation of the
anteroposterior cutting block if the posterior condyler line is used for rotational alignment. The medial
soft tissues are typically attenuated and stretched.

RECOMMENDED READINGS
Favorito PJ, Mihalko WM, Krackow KA. Total knee arthroplasty in the valgus knee. J Am Acad Orthop
Surg. 2002 Jan-Feb;10(1):16-24. Review. PubMed PMID: 11809047.
McAuley JP, Collier MB, Hamilton WG, Tabaraee E, Engh GA. Posterior cruciate-retaining total knee
arthroplasty for valgus osteoarthritis. Clin Orthop Relat Res. 2008 Nov;466(11):2644-9. Epub 2008 Aug
19. PubMed PMID: 18712454.

© 2013 American Academy of Orthopaedic Surgeons 2013 Adult Reconstructive Surgery of the Hip and Knee Self-Assessment Examination
46 • American Academy of Orthopaedic Surgeons

CLINICAL SITUATION FOR QUESTIONS 41 AND 42


A 72-year-old man with previous total hip arthroplasty developed hip pain of 1 month’s duration. He
underwent dental work 6 weeks ago. Aspiration showed a white blood cell count of > 6000 cells/µL
(reference range, 4500-11000/µL) and presence of gram-positive cocci in clusters on gram stain. The
orthopaedic surgeon recommended urgent debridement and irrigation. The components were judged to be
stable with regard to fixation, and the surgeon elected to retain the implants.

Question 41
What is this patient’s prognosis for infection resolution?

1. Good because it is a gram-positive organism


2. Good because it is an acute infection
3. Poor because it is a gram-positive organism
4. Poor because it is a late infection

PREFERRED RESPONSE: 4

Question 42
The patient had a final culture that revealed methicillin-resistant Staphylococcus aureus (MRSA). If the
attending physician recommended the 2-stage protocol including the use of an antibiotic-cement spacer,
what is the most likely prognosis for this patient?

1. Better functional outcome in comparison to infections from sensitive organisms


2. Same functional outcome as infections from sensitive organisms
3. Same prognosis for eradication of infection as infections from sensitive organisms
4. Poorer prognosis for eradication of infection compared to infection from sensitive organisms

PREFERRED RESPONSE: 4

DISCUSSION FOR QUESTIONS 41 AND 42


This is a late infection of at least 4 weeks symptomatic duration that most probably is hematogenous in
etiology. This is not an acute hematogenous infection that can successfully be treated with irrigation
and debridement. Retention of the implants with debridement and irrigation alone has been associated
with poor prognosis. In a recent study, the success rate was only 44% in a series of 104 patients at a
mean 5.7 years of follow-up. In one study of 50 infections attributable to MRSA or methicillin-resistant
Staphylococcus epidermidis organisms treated with a 2-stage protocol, the failure rate was 21%. Patients
who experienced successful infection treatment had lower functional outcome measures using the
WOMAC (Western Ontario and McMaster Universities Osteoarthritis Index), UCLA (University of
California Los Angeles) activity, and Oxford-12 scores, however.

© 2013 American Academy of Orthopaedic Surgeons 2013 Adult Reconstructive Surgery of the Hip and Knee Self-Assessment Examination
2013 Adult Reconstructive Surgery of the Hip and Knee Self-Assessment Examination Answer Book • 47

RECOMMENDED READINGS FOR QUESTIONS 41 AND 42


Azzam KA, Seeley M, Ghanem E, Austin MS, Purtill JJ, Parvizi J. Irrigation and debridement in
the management of prosthetic joint infection: traditional indications revisited. J Arthroplasty. 2010
Oct;25(7):1022-7. Epub 2010 Apr 8. PubMed PMID: 20378306.
Marculescu CE, Berbari EF, Hanssen AD, Steckelberg JM, Harmsen SW, Mandrekar JN, Osmon DR.
Outcome of prosthetic joint infections treated with debridement and retention of components. Clin Infect
Dis. 2006 Feb 15;42(4):471-8. Epub 2006 Jan 5. PubMed PMID: 16421790.
Leung F, Richards CJ, Garbuz DS, Masri BA, Duncan CP. Two-stage total hip arthroplasty: how often
does it control methicillin-resistant infection? Clin Orthop Relat Res. 2011 Apr;469(4):1009-15. PubMed
PMID: 21161741.
Parvizi J, Pawasarat IM, Azzam KA, Joshi A, Hansen EN, Bozic KJ. Periprosthetic joint infection: the
economic impact of methicillin-resistant infections. J Arthroplasty. 2010 Sep;25(6 Suppl):103-7. Epub
2010 May 31. PubMed PMID: 20570103.

END OF SERIES

Question 43
A 59-year-old active woman underwent elective total hip replacement using a posterior approach. She had
minimal pain and was discharged to home 2 days after surgery. Four weeks later she dislocated her hip
while shaving her legs. She underwent a closed reduction in the emergency department. Postreduction
radiographs show a reduced hip with well-fixed components in satisfactory alignment. What is the most
appropriate management of this condition from this point forward?

1. Observation and patient education regarding hip dislocation precautions


2. Revision to a larger-diameter femoral head
3. Revision to a constrained acetabular component
4. Application of a hip orthosis for 3 months

PREFERRED RESPONSE: 1

DISCUSSION
First-time early dislocations are often successfully treated without revision surgery, especially when
there is no component malalignment. In this clinical scenario, it appears the patient would benefit from
better education about dislocation precautions. Hip orthoses are of questionable benefit unless the patient
is cognitively impaired. Revision surgery can be successful, but is usually reserved for patients with
recurrent dislocations.

RECOMMENDED READINGS
Masri BA, Davidson D, Duncan CP, et al. Total hip arthroplasty complications. In: Barrack RL, Booth RE
Jr, Lonner JH, McCarthy JC, Mont MA, Rubash HE, eds. Orthopaedic Knowledge Update: Hip and Knee
Reconstruction 3. Rosemont, IL: American Academy of Orthopaedic Surgeons; 2006:475-503.
Dewal H, Maurer SL, Tsai P, Su E, Hiebert R, Di Cesare PE. Efficacy of abduction bracing in the
management of total hip arthroplasty dislocation. J Arthroplasty. 2004 Sep;19(6):733-8. PubMed PMID:
15343533.

© 2013 American Academy of Orthopaedic Surgeons 2013 Adult Reconstructive Surgery of the Hip and Knee Self-Assessment Examination
48 • American Academy of Orthopaedic Surgeons

Question 44
Patellar pain, subluxation, or dislocation after total knee arthroplasty can result from which of the
following component orientations?

1. Internal rotation of the tibial component


2. Lateralization of the tibial component
3. Lateralization of the femoral component
4. External rotation of the femoral component

PREFERRED RESPONSE: 1

DISCUSSION
Internal rotation of the components of a total knee arthroplasty, both the tibial and femoral components,
can lead to symptoms ranging from patellar pain to dislocation. Most researchers agree that proper
external rotation of the femoral component is parallel or nearly so to the femoral epicondylar axis with
the knee in the 90-degree flexed position. Proper rotational positioning of the tibial component places the
midportion of the tibial component rotationally aligned within the medial one-third of the tibial tubercle.
Internal rotation of the tibial component causes relative lateralization of the tibial tubercle and the extensor
mechanism. Lateralization of the femoral component moves the trochlear groove laterally. Lateralization
of the tibial component moves the tibial tubercle medially, which may be beneficial to patellar tracking.

RECOMMENDED READINGS
Berger RA, Crossett LS, Jacobs JJ, Rubash HE. Malrotation causing patellofemoral complications after
total knee arthroplasty. Clin Orthop Relat Res. 1998 Nov;(356):144-53. PubMed PMID: 9917679.
Barrack RL, Schrader T, Bertot AJ, Wolfe MW, Myers L. Component rotation and anterior knee pain after
total knee arthroplasty. Clin Orthop Relat Res. 2001 Nov;(392):46-55. PubMed PMID: 11716424.

Question 45
How does the risk for periprosthetic infection after total knee arthroplasty compare to risk for infection
after total hip arthroplasty?

1. Higher in primary arthroplasty


2. Lower in primary arthroplasty
3. Lower in revision arthroplasty
4. Equivalent in both primary and revision arthroplasty

PREFERRED RESPONSE: 1

DISCUSSION
Risk for periprosthetic infection is higher in the knee (1%-2%) than it is in the hip (0.3%-1.3%). The risk
for infection is higher after revision joint replacement surgery compared to primary joint replacement
surgery. Osteoarthritis is not associated with a higher risk for periprosthetic infection, but certain
inflammatory conditions such as rheumatoid arthritis and psoriatic arthritis place patients at higher risk for
postoperative infection.

© 2013 American Academy of Orthopaedic Surgeons 2013 Adult Reconstructive Surgery of the Hip and Knee Self-Assessment Examination
2013 Adult Reconstructive Surgery of the Hip and Knee Self-Assessment Examination Answer Book • 49

RECOMMENDED READINGS
Parvizi J. Periprosthetic joint infections. In: Lieberman JR, ed. AAOS Comprehensive Orthopaedic
Review. Vol 2. Rosemont, IL: American Academy of Orthopaedic Surgeons; 2009:1067-1073.
Masri BA, Davidson D, Duncan CP, et al. Total hip arthroplasty complications. In: Barrack RL, Booth RE
Jr, Lonner JH, McCarthy JC, Mont MA, Rubash HE, eds. Orthopaedic Knowledge Update: Hip and Knee
Reconstruction 3. Rosemont, IL: American Academy of Orthopaedic Surgeons; 2006:475-503.

Question 46
What factor is associated with a higher risk for dislocation after total hip arthroplasty?

1. Male gender
2. Previous hip surgery
3. A direct lateral surgical approach
4. Metal-on-metal bearing surfaces

PREFERRED RESPONSE: 2

DISCUSSION
Dislocation after total hip arthroplasty is a multifactorial problem. Numerous risk factors may act
independently or cumulatively to increase risk for this complication. Previous hip surgery of any kind
is associated with a twofold increased risk for dislocation. Other risk factors include female gender,
impaired mental status, inflammatory arthritis, and older age. Numerous studies have shown a lower
dislocation rate with a direct lateral approach, although surgical techniques such as capsular repair have
significantly lowered the incidence of dislocation after using the posterior approach. Metal-on-metal
bearings have been associated with other complications such as adverse tissue reactions but are often used
with larger-diameter bearings, which pose lower risk for dislocation.

RECOMMENDED READINGS
Masri BA, Davidson D, Duncan CP, et al. Total hip arthroplasty complications. In: Barrack RL, Booth RE
Jr, Lonner JH, McCarthy JC, Mont MA, Rubash HE, eds. Orthopaedic Knowledge Update: Hip and Knee
Reconstruction 3. Rosemont, IL: American Academy of Orthopaedic Surgeons; 2006:475-503.
Masonis JL, Bourne RB. Surgical approach, abductor function, and total hip arthroplasty dislocation. Clin
Orthop Relat Res. 2002 Dec;(405):46-53. Review. PubMed PMID: 12461355.

© 2013 American Academy of Orthopaedic Surgeons 2013 Adult Reconstructive Surgery of the Hip and Knee Self-Assessment Examination
50 • American Academy of Orthopaedic Surgeons

Question 47
What surgical technique has been associated with increased risk for recurrent dislocation after revision
total hip arthroplasty?

1. Posterior capsulorrhaphy
2. Use of a jumbo cup
3. Use of a lateralized liner
4. Use of a larger femoral head diameter

PREFERRED RESPONSE: 2

DISCUSSION
When addressing recurrent dislocation after total hip arthroplasty, surgical considerations that must be
addressed include approach, soft-tissue tension, component positioning, impingement, head size, and
acetabular liner profile. These considerations most often involve tensioning or augmentation of soft
tissues, as in capsulorrhaphy or trochanteric advancement; correction of malpositioned components; use
of larger femoral head sizes that increase motion before impingement; improving the head-to-neck ratio;
and increasing femoral offset. The use of a larger-diameter acetabular component may lead to soft-tissue
overgrowth around the liner, causing impingement and increasing the risk for recurrent dislocation.

RECOMMENDED READINGS
Soong M, Rubash HE, Macaulay W. Dislocation after total hip arthroplasty. J Am Acad Orthop Surg. 2004
Sep-Oct;12(5):314-21. Review. PubMed PMID: 15469226.
Jameson SS, Lees D, James P, Serrano-Pedraza I, Partington PF, Muller SD, Meek RM, Reed MR.
Lower rates of dislocation with increased femoral head size after primary total hip replacement: a five-
year analysis of NHS patients in England. J Bone Joint Surg Br. 2011 Jul;93(7):876-80. PubMed PMID:
21705556.

Question 48
Viscosupplementation (hyaluronan) achieves what effect in treatment of osteoarthritis pain of the knee?

1. Alters the natural history of osteoarthritis


2. No improvement in validated outcomes
3. Rebuilds articular cartilage
4. Beneficial for treating early-to-moderate osteoarthritis

PREFERRED RESPONSE: 4

DISCUSSION
Hyaluronan is a high-molecular-weight polysaccharide. Hyaluronan has been shown effective in treating
patients with early-to-moderate osteoarthritis. Hyaluronan products differ in method of production,
molecular weight biologic characteristics, and pharmacologic properties. Hyaluronan has not been shown
to rebuild articular cartilage or alter the natural history of osteoarthritis.

© 2013 American Academy of Orthopaedic Surgeons 2013 Adult Reconstructive Surgery of the Hip and Knee Self-Assessment Examination
2013 Adult Reconstructive Surgery of the Hip and Knee Self-Assessment Examination Answer Book • 51

RECOMMENDED READINGS
Divine JG, Zazulak BT, Hewett TE. Viscosupplementation for knee osteoarthritis: a systematic review.
Clin Orthop Relat Res. 2007 Feb;455:113-22. Review. Erratum in: Clin Orthop Relat Res. 2007
Jun;459:283. PubMed PMID: 17159579.
Richmond J, Hunter D, Irrgang J, Jones MH, Levy B, Marx R, Snyder-Mackler L, Watters WC 3rd,
Haralson RH 3rd, Turkelson CM, Wies JL, Boyer KM, Anderson S, St Andre J, Sluka P, McGowan R;
American Academy of Orthopaedic Surgeons. Treatment of osteoarthritis of the knee (nonarthroplasty). J
Am Acad Orthop Surg. 2009 Sep;17(9):591-600. PubMed PMID: 19726743.

CLINICAL SITUATION FOR QUESTIONS 49 AND 50


A 40-year-old man with a history of Legg-Calve-Perthes disease underwent a right hip resurfacing 3 years
ago with no perioperative complications. He has developed hip pain gradually during the last 4 months.
Radiographs show no evidence of fixation loosening or any adverse changes at the femoral neck. There is
no periarticular osteolysis.

Question 49
What is the most appropriate management of this condition?

1. Continue to observe with repeat radiographs in 6 months.


2. Fluoroscopic-guided iliopsoas tendon cortisone injection
3. Hip aspiration
4. Serum cobalt and chromium levels and metal-reduction MRI scan

PREFERRED RESPONSE: 4

Question 50
The patient developed a large intra-articular and intrapelvic pseudotumor. What predominant histological
features are present in such a lesion?

1. Polymorphonuclear leukocytes
2. Extracellular metal-wear debris
3. Cement particles within the macrophages
4. Lymphocytes and plasma cells

PREFERRED RESPONSE: 4

© 2013 American Academy of Orthopaedic Surgeons 2013 Adult Reconstructive Surgery of the Hip and Knee Self-Assessment Examination
52 • American Academy of Orthopaedic Surgeons

DISCUSSION FOR QUESTIONS 49 AND 50


Controversies remain with regard to the best approach for treating patients with metal-on-metal (MOM)
hip arthroplasties. All patients with painful MOM hip arthroplasties should be examined for fixation
loosening, wear/osteolysis, and infection no differently than patients with non-MOM hip arthroplasties.
Obtaining serum trace element levels is recommended. If the levels are high, cross-sectional imaging
should be obtained to determine if there is any pseudotumor or tissue necrosis around the hip arthroplasty.
Hip aspiration should be considered if there is concern for infection and if erythrocyte sedimentation
rate and C-reactive protein are elevated. Aseptic lymphocytic vasculitis-associated lesions have been
identified as occurring around MOM hip arthroplasties. Histologic features are predominantly tissue
necrosis with infiltration of lymphocytes and plasma cells.

RECOMMENDED READINGS FOR QUESTIONS 49 AND 50


Skinner J, Gie G, Kay P. British Orthopaedic Association. Metal on Metal Hip Replacement and Hip
Resurfacing Arthroplasty: What Does the MHRA Medical Device Alert Mean? British Hip Society, April
2010. http://www.chirurgenarnhem.nl/media/File/orthopedie/Aandoening%20pagina%20orthopedie/
MoM_BOA-BHS_AdvicetoSurgeons_2[1].pdf (Accessed 8/21/2012).
American Academy of Orthopaedic Surgeons: Metal-on-Metal Hip Replacement. Rosemont, IL: American
Academy of Orthopaedic Surgeons, 10/29/2010. http://www6.aaos.org/news/pemr/releases/release.
cfm?releasenum=934 (Accessed 8/21/2012).
McGrory B, Barrack R, Lachiewicz PF, Schmalzried TP, Yates AJ Jr, Watters WC 3rd, Turkelson
CM, Wies JL, St Andre J. Modern metal-on-metal hip resurfacing. J Am Acad Orthop Surg. 2010
May;18(5):306-14. Review. PubMed PMID: 20435881.
Glyn-Jones S, Pandit H, Kwon YM, Doll H, Gill HS, Murray DW. Risk factors for inflammatory
pseudotumour formation following hip resurfacing. J Bone Joint Surg Br. 2009 Dec;91(12):1566-74.
PubMed PMID: 19949118.

END OF SERIES

Question 51
A 72-year-old man was scheduled for left total knee replacement. He has a history of hypertension
and deep venous thrombosis (DVT) in his right lower extremity after an ankle fracture 2 years ago that
was treated nonsurgically. The patient asked about the recommended types of DVT prophylaxis or
investigations. Based on the 2011 AAOS Clinical Practice Guideline, Preventing Venous Thromboembolic
Disease in Patients Undergoing Elective Hip and Knee Arthroplasty, what is an acceptable option?

1. Six weeks of acetylsalicylic acid postsurgically beginning the evening of surgery


2. Six weeks of low-molecular-weight heparin beginning the morning after surgery
3. Routine duplex scans of both lower extremities before hospital discharge to ensure the patient
has not developed another DVT
4. Use of pneumatic calf compressors on both lower extremities while in the hospital and 4
weeks of warfarin starting the evening of surgery

PREFERRED RESPONSE: 4

© 2013 American Academy of Orthopaedic Surgeons 2013 Adult Reconstructive Surgery of the Hip and Knee Self-Assessment Examination
2013 Adult Reconstructive Surgery of the Hip and Knee Self-Assessment Examination Answer Book • 53

DISCUSSION
The 2011 AAOS guideline, Preventing Venous Thromboembolic Disease in Patients Undergoing
Elective Hip and Knee Arthroplasty, recommends the combined use of mechanical and pharmacological
prophylaxis in patients who have a history of previous thromboembolism. This recommendation is a
consensus opinion of the work group that established these guidelines because there is no other reliable
evidence for this clinical scenario. There is strong evidence against the use of routine duplex scans in
patients undergoing hip and knee replacement. The remaining two responses are less appropriate because
they do not include the use of mechanical prophylaxis immediately after surgery.

RECOMMENDED READINGS
American Academy of Orthopaedic Surgeons: Preventing Venous Thromboembolic Disease in Patients
Undergoing Elective Hip and Knee Arthroplasty. Rosemont, IL: American Academy of Orthopaedic
Surgeons, September 2011. Available at http://www.aaos.org/research/guidelines/VTE/VTE_guideline.asp
Accessed 8/21/2012.

Question 52
What has been identified as a risk factor for total knee arthroplasty failure after previous high tibial
osteotomy?

1. Body mass index higher than 35


2. Female gender
3. Preoperative stiffness
4. Advanced age

PREFERRED RESPONSE: 1

DISCUSSION
Increased weight, male gender, young age at the time of total knee arthroplasty, laxity, and limb
malalignment preoperatively have been identified as risk factors for early failure for total knee arthroplasty
following high tibial osteotomy.

RECOMMENDED READINGS
Parvizi J, Hanssen AD, Spangehl MJ. Total knee arthroplasty following proximal tibial osteotomy: risk
factors for failure. J Bone Joint Surg Am. 2004 Mar;86-A(3):474-9. PubMed PMID: 14996871.
Akizuki S, Shibakawa A, Takizawa T, Yamazaki I, Horiuchi H. The long-term outcome of high tibial
osteotomy: a ten- to 20-year follow-up. J Bone Joint Surg Br. 2008 May;90(5):592-6. PubMed PMID:
18450624.

© 2013 American Academy of Orthopaedic Surgeons 2013 Adult Reconstructive Surgery of the Hip and Knee Self-Assessment Examination
54 • American Academy of Orthopaedic Surgeons

Question 53
What limits indications for the use of constrained liners?

1. Association with periprosthetic fracture


2. Technical difficulty associated with insertion
3. High costs associated with their use
4. High failure rates associated with their use

PREFERRED RESPONSE: 4

DISCUSSION
Because of reports of relatively high failure rates associated with constrained liners, indications are
limited to continued instability after appropriate component position or deficient abductor mechanism
and instability. Neither cost nor technical insertion issues are relevant with regard to indications for use.
Periprosthetic fractures are not associated with constrained liner usage.

RECOMMENDED READINGS
Carter AH, Sheehan EC, Mortazavi SM, Purtill JJ, Sharkey PF, Parvizi J. Revision for recurrent instability:
what are the predictors of failure? J Arthroplasty. 2011 Sep;26(6 Suppl):46-52. Epub 2011 May 8. PubMed
PMID: 21550768.
Della Valle CJ, Chang D, Sporer S, Berger RA, Rosenberg AG, Paprosky WG. High failure rate of a
constrained acetabular liner in revision total hip arthroplasty. J Arthroplasty. 2005 Oct;20(7 Suppl 3):103-
7. PubMed PMID: 16214010.

Question 54
A 70-year-old woman with a body mass index (BMI) of 34 and a history of hypercholesterolemia has
elected to undergo total hip arthroplasty. Her son recently learned he has Factor V Leiden following an
episode of pulmonary embolism. What are this patient’s risk factors for thromboembolic disease?

1. Type of surgery, age, and BMI


2. Type of surgery, hypercholesterolemia, and age
3. Age, BMI, and hypercholesterolemia
4. BMI, type of surgery, and hypercholesterolemia

PREFERRED RESPONSE: 1

DISCUSSION
Risk stratification is one of the most critical clinical evaluations before undergoing total joint arthroplasty.
Many factors have been identified to increase risk for venous thromboembolism (VTE). The major factors
include previous VTE, obesity, type of surgery (such as total joint arthroplasty), hypercoagulable states,
myocardial infarction, congestive heart failure, family history of VTE, and hormonal replacement therapy.
Hypercholesterolemia is not a risk factor for thromboembolic disease.

© 2013 American Academy of Orthopaedic Surgeons 2013 Adult Reconstructive Surgery of the Hip and Knee Self-Assessment Examination
2013 Adult Reconstructive Surgery of the Hip and Knee Self-Assessment Examination Answer Book • 55

RECOMMENDED READINGS
Geerts WH, Bergqvist D, Pineo GF, Heit JA, Samama CM, Lassen MR, Colwell CW; American College
of Chest Physicians. Prevention of venous thromboembolism: American College of Chest Physicians
Evidence-Based Clinical Practice Guidelines (8th Edition). Chest. 2008 Jun;133(6 Suppl):381S-453S.
PubMed PMID: 18574271.
Caprini JA, Tapson VF, Hyers TM, Waldo AL, Wittkowsky AK, Friedman R, Colgan KJ, Shillington AC;
NABOR Steering Committee. Treatment of venous thromboembolism: adherence to guidelines and impact
of physician knowledge, attitudes, and beliefs. J Vasc Surg. 2005 Oct;42(4):726-33. PubMed PMID:
16242561.

CLINICAL SITUATION FOR QUESTIONS 55 AND 56


An otherwise healthy 79-year-old man underwent a total hip arthroplasty 5 years ago. He has had a 48-
hour history of groin and thigh pain and malaise. Examination reveals pain with internal motion of the
hip. Radiographs show well-fixed, appropriately positioned components.

Question 55
What serum inflammatory marker has the highest correlation with periprosthetic joint infection?

1. C-reactive protein
2. Serum white blood cell count
3. Erythrocyte sedimentation rate
4. Interleukin 6 (IL-6)

PREFERRED RESPONSE: 4

Question 56
Serum blood work reveals markedly elevated erythrocyte sedimentation rate (ESR), C-reactive protein
(CRP), and IL-6 levels. Cultures from a hip aspirate reveal a low virulence staph epidermis. What is the
next appropriate step in management?

1. Arthroscopic debridement
2. Reaspiration to confirm that the organism is not a contaminant
3. Indium scan to evaluate for infection
4. Irrigation and debridement and head and liner exchange

PREFERRED RESPONSE: 4

© 2013 American Academy of Orthopaedic Surgeons 2013 Adult Reconstructive Surgery of the Hip and Knee Self-Assessment Examination
56 • American Academy of Orthopaedic Surgeons

DISCUSSION FOR QUESTIONS 55 AND 56


Although CRP and ESR can be elevated in the setting of infection, IL-6 has been shown to have the
highest correlation with infection. Serum white blood cell count has been shown to be ineffective in
correlating with periprosthetic joint infection. Given the history, it is likely this patient has an acute
hematogenous infection. Appropriate steps including initiation of intravenous antibiotics after cultures
have been obtained and medical optimization with treatment consisting of irrigation and debridement with
head and liner exchange. There is no role for an indium scan or a repeat aspiration in the presence of an
elevated ESR, CRP, and IL-6. There is no role for arthroscopic debridement in this case.

RECOMMENDED READINGS FOR QUESTIONS 55 AND 56


Berbari E, Mabry T, Tsaras G, Spangehl M, Erwin PJ, Murad MH, Steckelberg J, Osmon D. Inflammatory
blood laboratory levels as markers of prosthetic joint infection: a systematic review and meta-analysis. J
Bone Joint Surg Am. 2010 Sep 1;92(11):2102-9. Review. PubMed PMID: 20810860.
Jacovides CL, Parvizi J, Adeli B, Jung KA. Molecular markers for diagnosis of periprosthetic joint
infection. J Arthroplasty. 2011 Sep;26(6 Suppl):99-103.e1. Epub 2011 May 13. PubMed PMID: 21570803.
Della Valle C, Parvizi J, Bauer TW, DiCesare PE, Evans RP, Segreti J, Spangehl M, Watters WC 3rd,
Keith M, Turkelson CM, Wies JL, Sluka P, Hitchcock K; American Academy of Orthopaedic Surgeons.
American Academy of Orthopaedic Surgeons clinical practice guideline on: the diagnosis of periprosthetic
joint infections of the hip and knee. J Bone Joint Surg Am. 2011 Jul 20;93(14):1355-7. PubMed PMID:
21792503.
Tsukayama DT, Estrada R, Gustilo RB. Infection after total hip arthroplasty. A study of the treatment of
one hundred and six infections. J Bone Joint Surg Am. 1996 Apr;78(4):512-23. PubMed PMID: 8609130.

END OF SERIES

Question 57
A 70-year-old man complains of symptomatic medial knee pain that has become progressively worse
during the past year. An MRI scan reveals a complex posterior horn medial meniscus tear with associated
medial and patellofemoral cartilage defects. Radiographs reveal medial joint space narrowing and
osteophytes in the other compartments. What treatment is most likely to provide long-term, durable
symptom relief?

1. High tibial osteotomy


2. Total knee replacement
3. Unicondylar knee replacement
4. Arthroscopic partial menisectomy

PREFERRED RESPONSE: 2

© 2013 American Academy of Orthopaedic Surgeons 2013 Adult Reconstructive Surgery of the Hip and Knee Self-Assessment Examination
2013 Adult Reconstructive Surgery of the Hip and Knee Self-Assessment Examination Answer Book • 57

DISCUSSION
Total knee replacement is a well-established surgery for diffuse, symptomatic osteoarthritis of the knee
joint, and its efficacy has been shown in many studies. According to the 2008 AAOS Clinical Practice
Guideline, Treatment of Osteoarthritis of the Knee, arthroscopy in the setting of existing osteoarthritis is
efficacious for relieving the signs and symptoms of a torn meniscus, but not osteoarthritis. Likewise, in
young and active patients, clinical outcomes show improvement after realignment osteotomy for single-
compartment osteoarthritis. Unicondylar knee replacement is not indicated for tricompartmental disease
of the knee.

RECOMMENDED READINGS
Brouwer RW, Bierma-Zeinstra SM, van Raaij TM, Verhaar JA. Osteotomy for medial compartment
arthritis of the knee using a closing wedge or an opening wedge controlled by a Puddu plate. A one-year
randomised, controlled study. J Bone Joint Surg Br. 2006 Nov;88(11):1454-9. PubMed PMID: 17075089.
W-Dahl A, Toksvig-Larsen S, Roos EM. A 2-year prospective study of patient-relevant outcomes in
patients operated on for knee osteoarthritis with tibial osteotomy. BMC Musculoskelet Disord. 2005 Apr
5;6:18. PubMed PMID: 15811186.

Question 58
A 68-year-old man reports hip pain 15 years after successful cementless total hip arthroplasty.
Radiographs show 3 mm of linear wear of the modular acetabular liner and a retro-acetabular osteolytic
lesion. Both the titanium femoral and acetabular components appear to be well fixed. The orthopaedic
surgeon recommends revision of the acetabular liner and femoral head. This patient is at increased risk for

1. dislocation.
2. periprosthetic fracture.
3. infection.
4. progressive osteolysis.

PREFERRED RESPONSE: 1

DISCUSSION
Isolated acetabular liner revision is frequently performed in cases of liner wear and periprosthetic
osteolysis in the absence of acetabular component loosening. Many reports have documented an increased
incidence of dislocation following this type of revision surgery. This dislocation rate can be reduced
by using a larger-diameter femoral head at the time of revision. If the acetabular component is loose
or malpositioned, it should be revised. If the locking mechanism is damaged, then a replacement liner
may be cemented into the well-fixed shell. Numerous studies have shown that many osteolytic lesions
will reduce in size or heal without bone grafting, and removal of the source of wear debris will arrest
the progression of osteolysis. The risk for periprosthetic fracture and infection are lower than risk for
dislocation in this setting.

© 2013 American Academy of Orthopaedic Surgeons 2013 Adult Reconstructive Surgery of the Hip and Knee Self-Assessment Examination
58 • American Academy of Orthopaedic Surgeons

RECOMMENDED READINGS
Grose A, Zelicof SB: Osteolysis. In: Barrack RL, Booth RE Jr, Lonner JH, McCarthy JC, Mont MA,
Rubash HE, eds. Orthopaedic Knowledge Update: Hip and Knee Reconstruction 3. Rosemont, IL:
American Academy of Orthopaedic Surgeons; 2006:521-528.
Boucher HR, Lynch C, Young AM, Engh CA Jr, Engh C Sr. Dislocation after polyethylene liner exchange
in total hip arthroplasty. J Arthroplasty. 2003 Aug;18(5):654-7. PubMed PMID: 12934221.
Maloney WJ, Paprosky W, Engh CA, Rubash H. Surgical treatment of pelvic osteolysis. Clin Orthop Relat
Res. 2001 Dec;(393):78-84. PubMed PMID: 11764374.

Question 59
A 61-year-old man with a body mass index of 31 had a 6-month gradual onset of right medial knee pain.
Examination revealed a small effusion, stable ligaments, a normally tracking patella, and mild medial joint
line tenderness. Standing radiographs show mild medial joint space narrowing. Effective treatment at this
stage of early medial compartmental osteoarthritis includes

1. glucosamine 1500 mg/day and chondroitin sulfate 800 mg/day.


2. weight loss through dietary management and low-impact aerobic exercises.
3. arthroscopic debridement and lavage.
4. a valgus-directing brace.

PREFERRED RESPONSE: 2

DISCUSSION
According to the 2008 AAOS Clinical Practice Guideline, Treatment of Osteoarthritis of the Knee
(Non-Arthroplasty), Level 1 evidence confirms that weight loss and exercise benefit patients with
knee osteoarthritis. The other responses have either inclusive evidence (a valgus-directing brace) or
no evidence to support their use (glucosamine 1500 mg/day and chondroitin sulfate 800 mg/day and
arthroscopic debridement and lavage).

RECOMMENDED READINGS
American Academy of Orthopaedic Surgeons: Treatment of Osteoarthritis of the Knee. Rosemont, IL:
American Academy of Orthopaedic Surgeons, December 2008. Available at http://www.aaos.org/research/
guidelines/GuidelineOAKnee.asp Accessed 8/21/2012.
Richmond J, Hunter D, Irrgang J, Jones MH, Levy B, Marx R, Snyder-Mackler L, Watters WC 3rd,
Haralson RH 3rd, Turkelson CM, Wies JL, Boyer KM, Anderson S, St Andre J, Sluka P, McGowan R;
American Academy of Orthopaedic Surgeons. Treatment of osteoarthritis of the knee (nonarthroplasty). J
Am Acad Orthop Surg. 2009 Sep;17(9):591-600. PubMed PMID: 19726743.

© 2013 American Academy of Orthopaedic Surgeons 2013 Adult Reconstructive Surgery of the Hip and Knee Self-Assessment Examination
2013 Adult Reconstructive Surgery of the Hip and Knee Self-Assessment Examination Answer Book • 59

Figure 60a Figure 60b Figure 60c Figure 60d

Figure 60e
Question 60
Figures 60a through 60c show the intraoperative photograph, anteroposterior radiograph, and axial MRI
scan of a 63-year-old man who had right groin pain 18 months after undergoing an uncemented right
total hip replacement using a modular femoral neck implant and a metal-on-polyethylene bearing. His
laboratory studies revealed an erythrocyte sedimentation rate of 8 mm/h (reference range, 0-20 mm/h) and
C-reactive protein level of 5.4 mg/L (reference range, 0.08-3.1 mg/L). A preoperative aspiration revealed
cultures that were negative for infection. A cell could not be obtained for evaluation because the cells
were “degenerative.” At the time of surgery the joint fluid was turbid in appearance; the periarticular
tissues appeared avascular and tan/beige in color. An intraoperative frozen section was negative for acute
inflammation. The implants were solidly fixed to bone. The cause of this patient’s symptoms and the
intraoperative findings most likely are attributable to

1. “backside” polyethylene wear.


2. metal debris.
3. soft-tissue sarcoma.
4. iliopsoas tendonitis.

PREFERRED RESPONSE: 2

DISCUSSION
Metal debris has caused an adverse local soft-tissue response in this scenario. The laboratory studies are
all negative for infection. The MRI scan shows a large soft-tissue mass that is posterior to the hip and
originating from the hip joint. The dysvascular appearance of the tissues is typical for an adverse tissue
response to metal. Even though the bearing is polyethylene, the modular junctions created sufficient metal
debris to cause the reaction. Figures 60d and 60e show metal debris inside of the removed femoral head
and corrosion of the modular neck. The other diagnoses are not consistent with this clinical presentation.

© 2013 American Academy of Orthopaedic Surgeons 2013 Adult Reconstructive Surgery of the Hip and Knee Self-Assessment Examination
60 • American Academy of Orthopaedic Surgeons

RECOMMENDED READINGS
Kop AM, Keogh C, Swarts E. Proximal component modularity in THA--at what cost? An implant retrieval
study. Clin Orthop Relat Res. 2012 Jul;470(7):1885-94. PubMed PMID: 22048865.
Nganbe M, Louati H, Khan U, Speirs A, Beaule PE. Retrieval analysis and in vitro assessment of
strength, durability, and distraction of a modular total hip replacement. J Biomed Mater Res A. 2010 Dec
1;95(3):819-27. PubMed PMID: 20737432.
Williams DH, Greidanus NV, Masri BA, Duncan CP, Garbuz DS. Prevalence of Pseudotumor in
Asymptomatic Patients After Metal-on-Metal Hip Arthroplasty. J Bone Joint Surg Am. 2011 Dec
7;93(23):2164-71. PubMed PMID: 22159851.
Kop AM, Swarts E. Corrosion of a hip stem with a modular neck taper junction: a retrieval study of 16
cases. J Arthroplasty. 2009 Oct;24(7):1019-23. Epub 2008 Oct 5. PubMed PMID: 18835686.

Question 61
The range of knee mobility after total knee replacement is multifactorial and dependent upon implant
design, surgical implantation accuracy, and patient-specific variables. What total knee implant design is
associated with the most knee flexion after total knee replacement?

1. Highly conforming articular surface geometry


2. Higher-flexion femoral component design manufactured to allow the most knee flexion
3. Posterior cruciate-stabilized implant, with or without a higher flexion manufacturing
modification
4. Posterior cruciate-retaining design with a mobile bearing, custom implanted based on CT scan
data

PREFERRED RESPONSE: 3

DISCUSSION
A posterior cruciate-stabilized implant has the best support in the literature in terms of the most favorable
range of motion after knee arthroplasty, regardless of whether the femoral component is designed with
a higher flexion variation. The higher flexion design is a manufacturing variation that is intended to
increase motion by clearing the posterior condyles in flexion. Although the knee may not gain more
flexion, this design allows for more safety in deep flexion. The long-term outcomes of increased stresses
on the polyethylene are not known, however. By itself, a higher-flexion design does not lead to increased
knee mobility. The effects of mobile bearings, custom CT scan-based knee implantation, and highly
conforming designs on ultimate knee range of motion are uncertain.

RECOMMENDED READINGS
Dennis DA, Komistek RD, Stiehl JB, Walker SA, Dennis KN. Range of motion after total knee
arthroplasty: the effect of implant design and weight-bearing conditions. J Arthroplasty. 1998
Oct;13(7):748-52. PubMed PMID: 9802659.
Dennis DA, Komistek RD, Mahfouz MR, Haas BD, Stiehl JB. Multicenter determination of in vivo
kinematics after total knee arthroplasty. Clin Orthop Relat Res. 2003 Nov;(416):37-57. PubMed PMID:
14646738.

© 2013 American Academy of Orthopaedic Surgeons 2013 Adult Reconstructive Surgery of the Hip and Knee Self-Assessment Examination
2013 Adult Reconstructive Surgery of the Hip and Knee Self-Assessment Examination Answer Book • 61

Question 62
In total knee arthroplasty, in vitro testing has shown that cross-linking can diminish the rate of
polyethylene wear by 30% to 80%. What other change in material properties is possible when
polyethylene is highly cross-linked?

1. Increased ductility
2. Increased wettability
3. Diminished fatigue strength
4. Decreased resistance to abrasive wear

PREFERRED RESPONSE: 3

DISCUSSION
The most important concern regarding highly cross-linked polyethylene relates to decreased mechanical
properties. Cross-linking results in reduced ductility, tensile strength, and fatigue crack propagation
resistance. These problems have not been identified as causing implant failure in most recent clinical
trials, but remain the most important mechanical issues associated with current material processing
methods.

RECOMMENDED READINGS
Fisher J, McEwen HM, Tipper JL, Galvin AL, Ingram J, Kamali A, Stone MH, Ingham E. Wear, debris,
and biologic activity of cross-linked polyethylene in the knee: benefits and potential concerns. Clin Orthop
Relat Res. 2004 Nov;(428):114-9. PubMed PMID: 15534530.
Muratoglu OK, Rubash HE, Bragdon CR, Burroughs BR, Huang A, Harris WH. Simulated normal gait
wear testing of a highly cross-linked polyethylene tibial insert. J Arthroplasty. 2007 Apr;22(3):435-44.
Erratum in: J Arthroplasty. 2008 Jan;23(1):158. PubMed PMID: 17400100.

Question 63
What factor is associated with a high risk for developing pseudotumors after metal-on-metal hip
resurfacing?

1. Large-diameter components
2. Age 40 or older for men
3. Age 40 or younger for women
4. Diagnosis of primary osteoarthritis

PREFERRED RESPONSE: 3

© 2013 American Academy of Orthopaedic Surgeons 2013 Adult Reconstructive Surgery of the Hip and Knee Self-Assessment Examination
62 • American Academy of Orthopaedic Surgeons

DISCUSSION
The recent experience of a large clinical cohort revealed the most likely risk factors as female gender, age
younger than 40, small components, and the diagnosis of hip dysplasia causing osteoarthritis. Failure was
least likely among men and procedures involving larger components. These data have prompted some
authors to caution against use in women and to primarily target candidates who are men younger than age
50. Small components may be more prone to failure because of malpositioning and edge loading, which
have been noted to be more common in dysplasia cases.

RECOMMENDED READINGS
Glyn-Jones S, Pandit H, Kwon YM, Doll H, Gill HS, Murray DW. Risk factors for inflammatory
pseudotumour formation following hip resurfacing. J Bone Joint Surg Br. 2009 Dec;91(12):1566-74.
PubMed PMID: 19949118.
Rieker CB, Schön R, Konrad R, Liebentritt G, Gnepf P, Shen M, Roberts P, Grigoris P. Influence of the
clearance on in-vitro tribology of large diameter metal-on-metal articulations pertaining to resurfacing hip
implants. Orthop Clin North Am. 2005 Apr;36(2):135-42, vii. PubMed PMID: 15833451.

Figure 64

CLINICAL SITUATION FOR QUESTIONS 64 THROUGH 66


Figure 64 is the radiograph of a 77-year-old woman with a painful total hip arthroplasty (THA) who
had surgery 15 years ago. Preoperative laboratory studies reveal a C-reactive protein (CRP) of 4 mg/L
(reference range, 0.08-3.1 mg/L). Her serum white blood cell (WBC) count and differential values are
within defined limits, and her erythrocyte sedimentation rate (ESR) is 35 mm/h (reference range, 0-20
mm/h).

© 2013 American Academy of Orthopaedic Surgeons 2013 Adult Reconstructive Surgery of the Hip and Knee Self-Assessment Examination
2013 Adult Reconstructive Surgery of the Hip and Knee Self-Assessment Examination Answer Book • 63

Question 64
What is the next appropriate step in management of the patient?

1. Labeled WBC scan


2. MRI scan of the hip to evaluate for fluid collection
3. Revision THA with gram stain and multiple cultures
4. Aspiration with cell count and differential and culture

PREFERRED RESPONSE: 4

Question 65
At the time of revision THA, the acetabular defect is confined to a contained cavitary defect in the
dome. Anterior and posterior columns are intact, as is the rim. What is the most appropriate acetabular
reconstruction?

1. Bilobed or “double bubble” acetabular component


2. Cementless acetabular hemisphere with multiple screws
3. Cemented all-polyethylene shell
4. Antiprotrusio cage device

PREFERRED RESPONSE: 2

Question 66
During the revision, a large anterior column defect secondary to reaming is noted. At the time of
impaction of the acetabular component, a loss of resistance is noted and the shell is unstable. After
removing the shell, a fracture through the posterior column is noted. What is the most appropriate course
of action?

1. Resection arthroplasty
2. Distraction of the fracture with a large porous acetabular component
3. Acetabular antiprotrusio cage with screw fixation
4. Revision of the acetabular component with allograft and a cemented socket

PREFERRED RESPONSE: 3

© 2013 American Academy of Orthopaedic Surgeons 2013 Adult Reconstructive Surgery of the Hip and Knee Self-Assessment Examination
64 • American Academy of Orthopaedic Surgeons

DISCUSSION FOR QUESTIONS 64 THROUGH 66


Aspiration is appropriate in the setting of a failed total joint replacement when ESR and CRP are elevated,
even with a normal systemic WBC count. A MRI scan of the hip is not useful in this setting. A labeled
WBC scan is of no additional value when an aspiration of the hip is planned. Although multiple cultures
should be obtained at the time of revision THA, preoperative aspiration is appropriate in this case. Most
acetabular revisions can be accomplished with a cementless hemispherical component. Supplemental
fixation with screws is indicated. In this patient, the acetabular rim and columns are intact and the first
choice is a cementless hemispherical component. Bilobed components have an unacceptably high failure
rate and are not indicated when the acetabular rim is intact. A cemented all-polyethylene shell is not
likely to achieve adequate long-term fixation with the sclerotic acetabular bone. An antiprotrusio device is
indicated in the setting of severe bone loss or pelvic discontinuity. An antiprotrusio cage is the best choice
in the setting of a pelvic discontinuity that has the capacity to heal. If the discontinuity does not have the
capacity to heal, it can be treated in distraction with an acetabular allograft supported by a cage or custom
triflange component. A large porous cup is not likely to achieve stable fixation and will require removal
of more acetabular bone. A cemented acetabular component will not achieve adequate stability or fixation
in the absence of additional support. Resection arthroplasty would not be the first choice in a potentially
reconstructable situation.

RECOMMENDED READINGS FOR QUESTIONS 64 THROUGH 66


Della Valle C, Parvizi J, Bauer TW, Dicesare PE, Evans RP, Segreti J, Spangehl M, Watters WC
3rd, Keith M, Turkelson CM, Wies JL, Sluka P, Hitchcock K; American Academy of Orthopaedic
Surgeons. Diagnosis of periprosthetic joint infections of the hip and knee. J Am Acad Orthop Surg. 2010
Dec;18(12):760-70. PubMed PMID: 21119142.
Claus AM, Engh CA Jr, Sychterz CJ, Xenos JS, Orishimo KF, Engh CA Sr. Radiographic definition of
pelvic osteolysis following total hip arthroplasty. J Bone Joint Surg Am. 2003 Aug;85-A(8):1519-26.
PubMed PMID: 12925632.
Sporer SM, Paprosky WG, O’Rourke MR. Managing bone loss in acetabular revision. Instr Course Lect.
2006;55:287-97. Review. PubMed PMID: 16958464.
Paprosky WG, Burnett RS. Assessment and classification of bone stock deficiency in revision total hip
arthroplasty. Am J Orthop (Belle Mead NJ). 2002 Aug;31(8):459-64. PubMed PMID: 12216969.
Paprosky WG, O’Rourke M, Sporer SM. The treatment of acetabular bone defects with an associated
pelvic discontinuity. Clin Orthop Relat Res. 2005 Dec;441:216-20. PubMed PMID: 16331006.

END OF SERIES

© 2013 American Academy of Orthopaedic Surgeons 2013 Adult Reconstructive Surgery of the Hip and Knee Self-Assessment Examination
2013 Adult Reconstructive Surgery of the Hip and Knee Self-Assessment Examination Answer Book • 65

Figure 67a Figure 67b

Question 67
Figure 67a is the radiograph of a 78-year-old woman who has a recent history of increasing thigh pain 12
years after undergoing total hip arthroplasty. Figure 67b is the radiograph after she fell and was unable to
ambulate. What is the most appropriate treatment?

1. Application of a femoral cable plate


2. Application of cerclage-wired double allograft femoral struts
3. Femoral revision with an uncemented long stem
4. Femoral revision with a cemented long-stem prosthesis

PREFERRED RESPONSE: 3

DISCUSSION
Surgical treatment of periprosthetic fractures of total hip replacement with a loose implant and progressive
bone loss is associated with a high complication rate. Recent literature would favor the use of long
“Wagner-type” stems that have a long distal taper that may optimally engage the remaining femoral shaft
isthmus. Plating options are problematic because the ability to use screws with the plate is limited by the
intramedullary stem. Use of long distally fixed stems circumvents this problem by enhancing fracture and
healing and creating a long-term prosthetic solution in these most difficult cases.

RECOMMENDED READINGS
Masri BA, Davidson D, Duncan CP, et al. Total hip arthroplasty complications. In: Barrack RL, Booth RE
Jr, Lonner JH, McCarthy JC, Mont MA, Rubash HE, eds. Orthopaedic Knowledge Update: Hip and Knee
Reconstruction 3. Rosemont, IL: American Academy of Orthopaedic Surgeons; 2006:475-503.
Kwong LM, Miller AJ, Lubinus P. A modular distal fixation option for proximal bone loss in revision total
hip arthroplasty: a 2- to 6-year follow-up study. J Arthroplasty. 2003 Apr;18(3 Suppl 1):94-7. PubMed
PMID: 12730939.

© 2013 American Academy of Orthopaedic Surgeons 2013 Adult Reconstructive Surgery of the Hip and Knee Self-Assessment Examination
66 • American Academy of Orthopaedic Surgeons

Question 68
What criterion is most reliable when attempting to establish a diagnosis of chronic periprosthetic joint
infection (PJI) of the hip and knee?

1. Positive bone scan


2. Elevated erythrocyte sedimentation rate (ESR)
3. Elevated serum white blood cell (WBC) count
4. Aspiration with > 2500 WBC per mm3

PREFERRED RESPONSE: 4

DISCUSSION
Aspiration with a cell count in the joint fluid that yields a WBC higher than 2500 per mm3 is consistent
with infection. ESR and C-reactive protein should be obtained when evaluating a patient for PJI. Joint
aspiration should be performed for patients being investigated for PJI when CRP and/or ESR values are
outside defined limits.

RECOMMENDED READINGS
Della Valle C, Parvizi J, Bauer TW, Dicesare PE, Evans RP, Segreti J, Spangehl M, Watters WC
3rd, Keith M, Turkelson CM, Wies JL, Sluka P, Hitchcock K; American Academy of Orthopaedic
Surgeons. Diagnosis of periprosthetic joint infections of the hip and knee. J Am Acad Orthop Surg. 2010
Dec;18(12):760-70. PubMed PMID: 21119142.
Parvizi J, Ghanem E, Azzam K, Davis E, Jaberi F, Hozack W. Periprosthetic infection: are current
treatment strategies adequate? Acta Orthop Belg. 2008 Dec;74(6):793-800. PubMed PMID: 19205327.

Question 69
A 70-year-old man is scheduled to undergo bearing surface revision for wear and osteolysis 10 years
after cementless total hip arthroplasty. The femoral head is 28 mm alumina-oxide ceramic material. The
components are in good position, and there is no evidence of fixation loosening of either component by
radiograph or preoperative bone scan. What outcome is associated with isolated polyethylene exchange?

1. Reduced risk for future wear and osteolysis with a larger femoral head
2. Reduced risk for future wear and osteolysis with a cobalt chrome femoral head
3. Similar risk for dislocation compared to primary total hip arthroplasty
4. Increased risk for dislocation compared to primary total hip arthroplasty

PREFERRED RESPONSE: 4

DISCUSSION
The major complication associated with polyethylene exchange is postoperative dislocation. Maloney
and associates noted a dislocation rate of 11% in a study of 35 hips after such revision. Boucher and
associates reported a 25% rate of dislocation in a study of 25 patients. Larger femoral heads result in
higher volumetric wear in contrast to smaller-diameter heads. Stem revision is not indicated because there
is no fixation loosening. Moreover, stem biomaterial has no effect on polyethylene wear.

© 2013 American Academy of Orthopaedic Surgeons 2013 Adult Reconstructive Surgery of the Hip and Knee Self-Assessment Examination
2013 Adult Reconstructive Surgery of the Hip and Knee Self-Assessment Examination Answer Book • 67

RECOMMENDED READINGS
Maloney WJ, Herzwurm P, Paprosky W, Rubash HE, Engh CA. Treatment of pelvic osteolysis associated
with a stable acetabular component inserted without cement as part of a total hip replacement. J Bone Joint
Surg Am. 1997 Nov;79(11):1628-34. PubMed PMID: 9384421.
Boucher HR, Lynch C, Young AM, Engh CA Jr, Engh C Sr. Dislocation after polyethylene liner exchange
in total hip arthroplasty. J Arthroplasty. 2003 Aug;18(5):654-7. PubMed PMID: 12934221.

Question 70
A healthy, active 68-year-old woman had a total hip arthroplasty 3 months ago. She has been to the
emergency department with a posterior dislocation 3 times during the last 2 months. Plain radiographs
and a CT scan confirm that the acetabular component is oriented in 5 degrees of retroversion and 55
degrees of abduction. What is the most appropriate treatment?

1. Revision of the femoral and acetabular components


2. Maximizing head-neck ratio and increasing head length
3. Acetabular component revision
4. Closed reduction with an abduction brace and reinforcement of hip precautions

PREFERRED RESPONSE: 3

DISCUSSION
Acetabular malposition can lead to recurrent instability. When this cause is confirmed, reorientation of
the component can lead to successful revision surgery. Revision of the femoral component may not be
necessary if the acetabular component is repositioned. Increasing length and maximizing head-neck ratio
cannot make up for component malposition. There is no role for nonsurgical treatment in the setting of
recurrent instability with component malposition in an active, healthy patient.

RECOMMENDED READINGS
Carter AH, Sheehan EC, Mortazavi SM, Purtill JJ, Sharkey PF, Parvizi J. Revision for recurrent instability:
what are the predictors of failure? J Arthroplasty. 2011 Sep;26(6 Suppl):46-52. Epub 2011 May 8. PubMed
PMID: 21550768.
Wera GD, Ting NT, Moric M, Paprosky WG, Sporer SM, Della Valle CJ. Classification and management
of the unstable total hip arthroplasty. J Arthroplasty. 2012 May;27(5):710-5. Epub 2011 Oct 29. PubMed
PMID: 22036933.

© 2013 American Academy of Orthopaedic Surgeons 2013 Adult Reconstructive Surgery of the Hip and Knee Self-Assessment Examination
68 • American Academy of Orthopaedic Surgeons

Question 71
A 55-year-old woman with history of HIV infection is scheduled for revision total knee arthroplasty to
address instability. The index surgery was done 3 years ago. What is the white blood cell (WBC) count
threshold in the synovial fluid for an infection diagnosis?

1. 100000
2. 25000
3. 2500
4. 250

PREFERRED RESPONSE: 3

DISCUSSION
Numerous centers have published data supporting the use of synovial fluid WBC level as one of the most
accurate diagnostic criterion for infection in total joint arthroplasty. The threshold ranges between 2000
and 3000 WBC/mL. The sensitivity and specificity have been demonstrated to be higher than 90%.

RECOMMENDED READINGS
Schinsky MF, Della Valle CJ, Sporer SM, Paprosky WG. Perioperative testing for joint infection in
patients undergoing revision total hip arthroplasty. J Bone Joint Surg Am. 2008 Sep;90(9):1869-75.
Erratum in: J Bone Joint Surg Am. 2010 Mar;92(3):707. PubMed PMID: 18762646.
Trampuz A, Hanssen AD, Osmon DR, Mandrekar J, Steckelberg JM, Patel R. Synovial fluid leukocyte
count and differential for the diagnosis of prosthetic knee infection. Am J Med. 2004 Oct 15;117(8):556-
62. PubMed PMID: 15465503.

Question 72
What is the most common complication after a total hip replacement done through the anterior (Smith-
Peterson) approach?

1. Lateral femoral cutaneous nerve injury


2. Heterotopic ossification
3. Femoral nerve palsy
4. Anterior dislocation

PREFERRED RESPONSE: 1

© 2013 American Academy of Orthopaedic Surgeons 2013 Adult Reconstructive Surgery of the Hip and Knee Self-Assessment Examination
2013 Adult Reconstructive Surgery of the Hip and Knee Self-Assessment Examination Answer Book • 69

DISCUSSION
Responses 2, 3, and 4 can occur after anterior-approach total hip replacement, but their incidence is
much lower than that of thigh numbness, which is associated with injury to the lateral femoral cutaneous
nerve. This injury is usually clinically inconsequential. Goulding and associates investigated the
incidence of lateral femoral cutaneous nerve injury after anterior hip replacement using self-administered
questionnaires to assess sensory deficits and various functional scores for up to 1 year after surgery.
Among patients, 81% reported some alteration in thigh sensation, but functional scores did not show any
limitations. Symptoms also decreased over time and only a small number of patients reported complete
resolution. Also, hip injections administered from the anterior direction risk injury to the lateral femoral
cutaneous nerve. Patients undergoing anterior total hip replacement should be counseled about this
specific risk before surgery.

RECOMMENDED READINGS
Goulding K, Beaule PE, Kim PR, Fazekas A. Incidence of lateral femoral cutaneous nerve neuropraxia
after anterior approach hip arthroplasty. Clin Orthop Relat Res. 2010 Sep;468(9):2397-404. PubMed
PMID: 20532717.
Leopold SS, Battista V, Oliverio JA. Safety and efficacy of intraarticular hip injection using anatomic
landmarks. Clin Orthop Relat Res. 2001 Oct;(391):192-7. PubMed PMID: 11603669.

© 2013 American Academy of Orthopaedic Surgeons 2013 Adult Reconstructive Surgery of the Hip and Knee Self-Assessment Examination
70 • American Academy of Orthopaedic Surgeons

Figure 73

Question 73
Figure 73 is the anteroposterior pelvis radiograph of a 58-year-old woman who reported chronic hip pain
and a clunking sensation 18 months after hip surgery. Laboratory test findings are negative for infection.
What is the most appropriate treatment?

1. Revision total hip arthroplasty


2. Trochanteric bursa injection
3. Acetabular component revision
4. A course of physical therapy

PREFERRED RESPONSE: 1

DISCUSSION
The radiograph shows a metal-on-metal hip resurfacing arthroplasty. The acetabular component is
malpositioned in excessive abduction, which may explain the clunking symptoms and be allowing the
joint to subluxate. The best option is to convert this failed surface replacement to a conventional total hip
arthroplasty. Metal-on-metal bearings pose risk for synovitis, metallosis, and adverse tissue reactions,
particularly when the acetabular component is malpositioned, resulting in edge loading (this patient’s
situation). Acetabular revision is possible, but would leave the patient with a metal-on-metal bearing and
higher risk for failure attributable to a metal debris reaction and a damaged femoral head component.

RECOMMENDED READINGS
De Haan R, Campbell PA, Su EP, De Smet KA. Revision of metal-on-metal resurfacing arthroplasty of the
hip: the influence of malpositioning of the components. J Bone Joint Surg Br. 2008 Sep;90(9):1158-63.
PubMed PMID: 18757954.
Amstutz HC, Le Duff MJ, Campbell PA, Gruen TA, Wisk LE. Clinical and radiographic results of
metal-on-metal hip resurfacing with a minimum ten-year follow-up. J Bone Joint Surg Am. 2010 Nov
17;92(16):2663-71. PubMed PMID: 21084576.

© 2013 American Academy of Orthopaedic Surgeons 2013 Adult Reconstructive Surgery of the Hip and Knee Self-Assessment Examination
2013 Adult Reconstructive Surgery of the Hip and Knee Self-Assessment Examination Answer Book • 71

Question 74
Early postoperative infections following primary total hip arthroplasty are most likely caused by which
organism?

1. Staphylococcus epidermidis
2. Streptococcus viridans
3. Propionibacterium acnes
4. Staphylococcus aureus

PREFERRED RESPONSE: 4

DISCUSSION
Staphylococcus aureus is the most common organism cultured in early (fewer than 4 weeks postoperative)
periprosthetic infections. Methicillin-resistant Staphylococcus aureus is becoming a more common
pathogen in certain patient populations. B-hemolytic Streptococcus and some gram-negative infections
can also be found in early postoperative infections. Staphylococcus epidermidis, Streptococcus viridans,
and Propionibacterium acnes are more commonly found in late (> 4 weeks postoperative) infections.

RECOMMENDED READINGS
Parvizi J. Periprosthetic joint infections. In: Lieberman JR, ed. AAOS Comprehensive Orthopaedic
Review. Vol 2. Rosemont, IL: American Academy of Orthopaedic Surgeons; 2009:1067-1073.
Masri BA, Davidson D, Duncan CP, et al. Total hip arthroplasty complications. In: Barrack RL, Booth RE
Jr, Lonner JH, McCarthy JC, Mont MA, Rubash HE, eds. Orthopaedic Knowledge Update: Hip and Knee
Reconstruction 3. Rosemont, IL: American Academy of Orthopaedic Surgeons; 2006:475-503.

© 2013 American Academy of Orthopaedic Surgeons 2013 Adult Reconstructive Surgery of the Hip and Knee Self-Assessment Examination
72 • American Academy of Orthopaedic Surgeons

Figure 75a Figure 75b Figure 75c

CLINICAL SITUATION FOR QUESTIONS 75 AND 76


Figures 75a through 75c are the radiographs and CT scan of a 58-year-old woman who underwent
cementless left total hip arthroplasty. Nine months after surgery, she continued to have groin pain when
she actively flexed her hip. She had trouble walking up stairs and getting out of her car.

Question 75
What is the most likely diagnosis?

1. Trochanteric bursitis
2. Femoral component loosening
3. Iliopsoas tendonitis
4. Acetabular component loosening

PREFERRED RESPONSE: 3

Question 76
This patient failed a course of appropriate nonsurgical treatment. What is the next step in definitive
treatment?

1. Acetabular component revision


2. Femoral component revision
3. Acetabular liner exchange
4. Trochanteric bursectomy

PREFERRED RESPONSE: 1

© 2013 American Academy of Orthopaedic Surgeons 2013 Adult Reconstructive Surgery of the Hip and Knee Self-Assessment Examination
2013 Adult Reconstructive Surgery of the Hip and Knee Self-Assessment Examination Answer Book • 73

DISCUSSION FOR QUESTIONS 75 AND 76


There are a number of possible causes of groin pain after total hip replacement, but an exact diagnosis may
remain elusive in some patients. Infection should be ruled out with laboratory studies and, if indicated,
diagnostic aspiration of the hip joint. Implant loosening should be evaluated by plain radiograph and bone
scan, if indicated. Synovitis resulting from wear debris should be considered in patients with polyethylene
liners who experience late-onset symptoms, or in any patient with a metal-on-metal bearing. This patient’s
symptoms are classic for iliopsoas tendonitis. Physical examination usually reveals pain and weakness
with resisted hip flexion. A cross-table lateral radiograph and CT scan show that the anterior edge of the
acetabulum protrudes beyond the anterior wall, thereby acting as a source of iliopsoas tendon irritation. In
such cases, acetabular component revision and repositioning is indicated. Fluoroscopic-guided iliopsoas
cortisone injection can help to establish the diagnosis and relieve groin pain. If the acetabular component
is well-positioned, then iliopsoas tenotomy should be considered.

RECOMMENDED READINGS FOR QUESTIONS 75 AND 76


Lachiewicz PF, Kauk JR. Anterior iliopsoas impingement and tendinitis after total hip arthroplasty. J Am
Acad Orthop Surg. 2009 Jun;17(6):337-44. Review. PubMed PMID: 19474443.
O’Sullivan M, Tai CC, Richards S, Skyrme AD, Walter WL, Walter WK. Iliopsoas tendonitis a
complication after total hip arthroplasty. J Arthroplasty. 2007 Feb;22(2):166-70. PubMed PMID:
17275628.

END OF SERIES

Question 77
A 72-year-old woman returns 3 weeks after a right total knee replacement. She has been experiencing
increasing pain, swelling, and decreasing range of motion during the last 10 days. Examination shows the
knee to be more swollen and warm than what is typical at 3 weeks after surgery. The knee feels stable,
but she has diffuse tenderness and range of motion is between 15 and 85 degrees. What is the most
appropriate investigation(s) to diagnose the etiology of her current problem?

1. Radiographs of the knee


2. Radiographs, erythrocyte sedimentation rate (ESR), and C-reactive protein (CRP)
3. Radiographs, ESR, CRP, and knee aspiration
4. No investigations are needed; reassure the patient that her findings are typical at this point

PREFERRED RESPONSE: 3

© 2013 American Academy of Orthopaedic Surgeons 2013 Adult Reconstructive Surgery of the Hip and Knee Self-Assessment Examination
74 • American Academy of Orthopaedic Surgeons

DISCUSSION
Although the differential diagnosis of her symptoms is broad at this point, the description of symptoms
and findings are concerning for postsurgical infection. Regardless of other possible causes, infection
needs to be investigated. This question is directed at the most appropriate investigations for the diagnosis
of deep infection in the early postsurgical time period. Radiographs should be obtained to rule out
mechanical causes. Although the CRP should be trending to normal by 3 to 4 weeks, ESR will likely
be elevated secondary to the surgery and difficult to interpret. However, baseline laboratory studies are
helpful in the event that follow-up laboratory studies are required. An aspiration for culture and cell
count should also be obtained; however, it should be recognized that the normal synovial cell count values
during the early postsurgical period (fewer than 6 weeks) are much higher than in the chronic setting. In
a study by Bedair and associates, the synovial white blood cell count (WBC) was found to be most useful
in predicting infection in patients fewer than 6 weeks after surgery. All but 1 of 19 infected patients
had a synovial WBC higher than or equal to 10700 cells/µl, and all but 1 of 127 noninfected knees had
a synovial WBC of < 27800 cells/µl. Mean synovial cell counts were 96000 (3520-570000) cells/µl
for the infected group, and 4200 (0-41000) cells/µl for the noninfected group. Using receiver operating
characteristic curves, these authors learned that the optimal synovial WBC cutoff was 27800 cells/µl,
giving 84% sensitivity, 99% specificity, and positive and negative predictive values of 94% and 98%,
respectively. Synovial cell differential was less useful with the optimal value of 89% neutrophils, giving a
sensitivity of 84% and specificity of 69%. In the chronic setting, a synovial WBC count of > ~3000 cells/
µl and percentage of neutrophils > 65% is considered suspicious for infection.

RECOMMENDED READINGS
Bedair H, Ting N, Jacovides C, Saxena A, Moric M, Parvizi J, Della Valle CJ. The Mark Coventry Award:
diagnosis of early postoperative TKA infection using synovial fluid analysis. Clin Orthop Relat Res. 2011
Jan;469(1):34-40. PubMed PMID: 20585914.
Mason JB, Fehring TK, Odum SM, Griffin WL, Nussman DS. The value of white blood cell counts before
revision total knee arthroplasty. J Arthroplasty. 2003 Dec;18(8):1038-43. Erratum in: J Arthroplasty. 2009
Dec;24(8):1293. PubMed PMID: 14658109.
Trampuz A, Hanssen AD, Osmon DR, Mandrekar J, Steckelberg JM, Patel R. Synovial fluid leukocyte
count and differential for the diagnosis of prosthetic knee infection. Am J Med. 2004 Oct 15;117(8):556-
62. PubMed PMID: 15465503.

Question 78
What is the most effective method to increase range of motion to impingement during total hip
arthroplasty?

1. Increase neck length with a skirted component


2. Increase femoral head diameter
3. Decrease femoral neck offset
4. Use an elevated lip liner

PREFERRED RESPONSE: 2

© 2013 American Academy of Orthopaedic Surgeons 2013 Adult Reconstructive Surgery of the Hip and Knee Self-Assessment Examination
2013 Adult Reconstructive Surgery of the Hip and Knee Self-Assessment Examination Answer Book • 75

DISCUSSION
Femoral heads with neck skirts impinge early and have less range of motion. Increased femoral neck
offset lengths have been shown to have better range of motion. Elevated lip liners increase the distance
the femoral head has to travel to become dislocated, but they reduce the overall range of motion to
impingement. Increasing the diameter of the femoral head when the diameter of the femoral neck remains
the same increases the range of motion to impingement and hip stability.

RECOMMENDED READINGS
Krenzel BA, Berend ME, Malinzak RA, Faris PM, Keating EM, Meding JB, Ritter MA. High preoperative
range of motion is a significant risk factor for dislocation in primary total hip arthroplasty. J Arthroplasty.
2010 Sep;25(6 Suppl):31-5. Epub 2010 Jun 11. PubMed PMID: 20541892.
Garbuz DS, Tannast M, Steppacher SD, Murphy SB, Sporer SM, Lavernia CJ. Complications of total hip
arthroplasty. In: Glassman AH, Lachiewicz PF, Tanzer M, eds. Orthopaedic Knowledge Update: Hip and
Knee Reconstruction 4. Rosemont, IL: American Academy of Orthopaedic Surgeons; 2011:343-362.

Question 79
What factor is associated with high wear and elevated serum metal ion levels after metal-on-metal
resurfacing hip arthroplasty?

1. Retroversion of the femoral component


2. Acetabular anteversion more than 20 degrees
3. Acetabular inclination more than 50 degrees
4. Recurrent dislocation of the hip prosthesis

PREFERRED RESPONSE: 3

DISCUSSION
Early failure of metal-on-metal hip resurfacing has been attributed to abnormal wear related to
impingement or malalignment. A high level of acetabular inclination over the Lewinnek safe zone of 50
degrees has been associated with high levels of chrome and cobalt ions. This may be related to the fact
that wear may be increased by the uncoverage of the implant, although evidence to prove this conclusion
is anecdotal. Additionally, periprosthetic fluid collections and soft-tissue masses have been found in these
cases. The other component positions would not necessarily be associated with impingement or wear.

RECOMMENDED READINGS
Hart AJ, Sabah S, Henckel J, Lewis A, Cobb J, Sampson B, Mitchell A, Skinner JA. The painful metal-on-
metal hip resurfacing. J Bone Joint Surg Br. 2009 Jun;91(6):738-44. PubMed PMID: 19483225.
Langton DJ, Jameson SS, Joyce TJ, Webb J, Nargol AV. The effect of component size and orientation
on the concentrations of metal ions after resurfacing arthroplasty of the hip. J Bone Joint Surg Br. 2008
Sep;90(9):1143-51. PubMed PMID: 18757952.

© 2013 American Academy of Orthopaedic Surgeons 2013 Adult Reconstructive Surgery of the Hip and Knee Self-Assessment Examination
76 • American Academy of Orthopaedic Surgeons

Figure 80a Figure 80b

Question 80
Figures 80a and 80b are the anteroposterior and lateral radiographs taken 8 months after revision of an
aseptically loose acetabular component of an 81-year-old man who has mild cognitive impairment but is
medically healthy. A 32-mm femoral head with metal-on-polyethylene was used at the time of revision.
He has a history of numerous subluxation events and 2 dislocations requiring closed reduction since the
cup revision. An abduction brace was prescribed after the first dislocation. The hip functions well apart
from the episodes of instability. Erythrocyte sedimentation rate and C-reactive protein values are within
defined limits. What is the most appropriate treatment for his recurrent instability?

1. Reinforce the use of an abduction brace when he is out of bed


2. Prescribe physical therapy focusing on abductor strengthening and reinforce hip precautions
3. Revision of the acetabular implant
4. Revision of the head and liner to a larger bearing size

PREFERRED RESPONSE: 4

DISCUSSION
This patient has had numerous episodes of instability and has difficulty remembering his position of risk.
He has tried an abduction brace without success. Surgery is indicated for this patient. The 32-mm head
appears small in the cup, indicating a very large acetabular component for which one could use a much
larger bearing. There is no need for cup revision because the implant is well positioned and not loose.

RECOMMENDED READINGS
Carter AH, Sheehan EC, Mortazavi SM, Purtill JJ, Sharkey PF, Parvizi J. Revision for recurrent instability:
what are the predictors of failure? J Arthroplasty. 2011 Sep;26(6 Suppl):46-52. Epub 2011 May 8. PubMed
PMID: 21550768.
Berry DJ, von Knoch M, Schleck CD, Harmsen WS. Effect of femoral head diameter and operative
approach on risk of dislocation after primary total hip arthroplasty. J Bone Joint Surg Am. 2005
Nov;87(11):2456-63. PubMed PMID: 16264121.

© 2013 American Academy of Orthopaedic Surgeons 2013 Adult Reconstructive Surgery of the Hip and Knee Self-Assessment Examination
2013 Adult Reconstructive Surgery of the Hip and Knee Self-Assessment Examination Answer Book • 77

Question 81
After undergoing elective knee arthroplasty, which risk factor is most likely to cause a symptomatic
pulmonary embolus?

1. Female gender
2. Previous knee surgery
3. High preadmission blood glucose
4. History of coronary artery disease

PREFERRED RESPONSE: 3

DISCUSSION
Diabetes mellitus and high blood glucose levels have been shown to increase risk for perioperative
complications and symptomatic pulmonary embolism in patients undergoing arthroplasty. Other risk
factors include a body mass index higher than 30 and bilateral knee arthroplasty. Neither coronary artery
disease, previous knee surgery, nor female gender has been shown to increase risk.

RECOMMENDED READINGS
Mraovic B, Hipszer BR, Epstein RH, Pequignot EC, Parvizi J, Joseph JI. Preadmission hyperglycemia
is an independent risk factor for in-hospital symptomatic pulmonary embolism after major orthopedic
surgery. J Arthroplasty. 2010 Jan;25(1):64-70. Epub 2008 Dec 4. PubMed PMID: 19056217.
Carr ME. Diabetes mellitus: a hypercoagulable state. J Diabetes Complications. 2001 Jan-
Feb;15(1):44-54. Review. PubMed PMID: 11259926.

Question 82
A 72-year-old patient fell 3 weeks after undergoing a total hip arthroplasty using cementless fixation of the
femoral component. She sustained a comminuted Vancouver type B-2 fracture with displacement of the
calcar fragment. What is the best treatment option?

1. Revision using a proximal femoral replacement prosthesis


2. Revision using a long-stem femoral prosthesis along with cerclage fixation
3. Open reduction internal fixation using a locking plate with strut graft
4. Protected weight bearing with abduction bracing

PREFERRED RESPONSE: 2

DISCUSSION
A Vancouver type B-2 periprosthetic fracture pattern (between the lesser trochanter and the tip of the
femoral component) frequently compromises stem fixation. This patient’s stem fixation status was not
stable because the stem was inserted only 3 weeks before the fracture occurred. The most effective
treatment for Vancouver type B-2 fractures is revision with a long stem to achieve distal diaphyseal
fixation, along with fixation of the fracture using cables or wires with or without strut graft based on the
quality and quantity of the bone stock.

© 2013 American Academy of Orthopaedic Surgeons 2013 Adult Reconstructive Surgery of the Hip and Knee Self-Assessment Examination
78 • American Academy of Orthopaedic Surgeons

RECOMMENDED READINGS
Masri BA, Davidson D, Duncan CP, et al. Total hip arthroplasty. In: Barrack RL, Booth RE Jr, Lonner
JH, McCarthy JC, Mont MA, Rubash HE, eds. Orthopaedic Knowledge Update: Hip and Knee
Reconstruction 3. Rosemont, IL: American Academy of Orthopaedic Surgeons; 2006:475-503.
Sledge JB 3rd, Abiri A. An algorithm for the treatment of Vancouver type B2 periprosthetic proximal
femoral fractures. J Arthroplasty. 2002 Oct;17(7):887-92. PubMed PMID: 12375248.

Figure 83a Figure 83b

Question 83
Figure 83a is the radiograph of a previously active patient with pain in her lower lumbar spine region and
lateral hip 6 months after a cementless hip arthroplasty. What is the most likely cause of this patient’s
symptoms?

1. Increased hip joint offset


2. Increased leg length
3. Increased hip joint offset and leg length
4. Increased leg length but no increase in offset

PREFERRED RESPONSE: 3

DISCUSSION
The patient has an approximate 1-cm increase in leg length and a 1-cm increase in offset (Figure 83b). A
transischial line intersecting the lesser trochanters shows a 1.14-cm difference in leg length. The lateral
distance from the pubic midline to the medial femoral neck shows a difference in lateral offset of 1.1 cm.
This has resulted in pelvic obliquity and an apparent leg length discrepancy in addition to a true leg length
discrepancy.

© 2013 American Academy of Orthopaedic Surgeons 2013 Adult Reconstructive Surgery of the Hip and Knee Self-Assessment Examination
2013 Adult Reconstructive Surgery of the Hip and Knee Self-Assessment Examination Answer Book • 79

RECOMMENDED READINGS
Barrack RL. Preoperative Care. In: Callaghan JJ, Rosenberg AG, Rubash HE, eds. The Adult Hip. Vol 2.
2nd ed. Philadelphia, PA: Lippincott Williams & Wilkins; 2007:884-910.
Incavo SJ, Havener T, Benson E, McGrory BJ, Coughlin KM, Beynnon BD. Efforts to improve
cementless femoral stems in THR: 2- to 5-year follow-up of a high-offset femoral stem with distal stem
modification (Secur-Fit Plus). J Arthroplasty. 2004 Jan;19(1):61-7. PubMed PMID: 14716653.

Question 84
In patients with Crowe types III and IV developmental dysplasia of the hip with high hip centers,
acetabular reconstruction often requires lowering the acetabular component into the native acetabulum. In
doing so, significant risk for limb lengthening beyond 4 cm exists, making the hip difficult to reduce and
also raising risk for nerve injury. Which technique is used to overcome this problem?

1. Subtrochanteric osteotomy with femoral shortening


2. Use of an offset femoral component
3. Use of a lateralized liner
4. Extended trochanteric osteotomy

PREFERRED RESPONSE: 1

DISCUSSION
When significant lengthening of a dysplastic hip will occur because a high dislocation is relocated into
a significantly lower acetabulum, a femoral shortening may be necessary to reduce the hip and avoid a
stretch injury to the sciatic nerve. No other choice specifically addresses the need for femoral shortening,
and high offset stems and lateralized liners may exacerbate the problem if used alone and without femoral
shortening.

RECOMMENDED READINGS
Masonis JL, Patel JV, Miu A, Bourne RB, McCalden R, Macdonald SJ, Rorabeck CH. Subtrochanteric
shortening and derotational osteotomy in primary total hip arthroplasty for patients with severe hip
dysplasia: 5-year follow-up. J Arthroplasty. 2003 Apr;18(3 Suppl 1):68-73. PubMed PMID: 12730932.
Krych AJ, Howard JL, Trousdale RT, Cabanela ME, Berry DJ. Total hip arthroplasty with shortening
subtrochanteric osteotomy in Crowe type-IV developmental dysplasia: surgical technique. J Bone Joint
Surg Am. 2010 Sep;92 Suppl 1 Pt 2:176-87. PubMed PMID: 20844173.

© 2013 American Academy of Orthopaedic Surgeons 2013 Adult Reconstructive Surgery of the Hip and Knee Self-Assessment Examination
80 • American Academy of Orthopaedic Surgeons

Figure 85a Figure 85b

Question 85
Figures 85a and 85b are the radiographs of an 81-year-old woman who is brought to the emergency
department after tripping and landing on her right knee. She had a right total knee replacement 8 years
before this injury. The replacement had functioned well, but now she cannot bear weight and examination
shows swelling and an abrasion over the patella. Neurovascular examination is unremarkable. What is
the most appropriate treatment?

1. Urgent open reduction and internal fixation using cerclage cables


2. Urgent closed, and possible open, reduction and internal fixation using a lateral periarticular
locking screw plate
3. Wait for the patellar abrasion to heal, and then proceed with revision of her knee replacement
to a distal femoral replacement.
4. Wait for the patellar abrasion to heal, and then proceed with revision of her femoral
component using a long-stemmed femoral revision stem.

PREFERRED RESPONSE: 2

DISCUSSION
This patient has a periprosthetic distal femoral fracture above a well-fixed femoral implant. This is a
long spiral fracture and revision of her implant is not required. The level of the fracture is suitable for
fixation. The most reliable fixation method is use of a locking screw plate. Cerclage fixation with cables
is insufficient for this fracture. Surgery does not need to be delayed for healing of the patellar abrasion
because the surgical approach is distant to the abrasion.

© 2013 American Academy of Orthopaedic Surgeons 2013 Adult Reconstructive Surgery of the Hip and Knee Self-Assessment Examination
2013 Adult Reconstructive Surgery of the Hip and Knee Self-Assessment Examination Answer Book • 81

RECOMMENDED READINGS
Kolb W, Guhlmann H, Windisch C, Marx F, Koller H, Kolb K. Fixation of periprosthetic femur fractures
above total knee arthroplasty with the less invasive stabilization system: a midterm follow-up study. J
Trauma. 2010 Sep;69(3):670-6. PubMed PMID: 20838138.
Haidukewych G, Sems SA, Huebner D, Horwitz D, Levy B. Results of polyaxial locked-plate fixation
of periarticular fractures of the knee. J Bone Joint Surg Am. 2007 Mar;89(3):614-20. PubMed PMID:
17332111.
Ricci WM, Loftus T, Cox C, Borrelli J. Locked plates combined with minimally invasive insertion
technique for the treatment of periprosthetic supracondylar femur fractures above a total knee arthroplasty.
J Orthop Trauma. 2006 Mar;20(3):190-6. PubMed PMID: 16648700.
Su ET, DeWal H, Di Cesare PE. Periprosthetic femoral fractures above total knee replacements. J Am
Acad Orthop Surg. 2004 Jan-Feb;12(1):12-20. Review. PubMed PMID: 14753793.

Question 86
A 65-year-old woman has had 6 months of activity-related pain in her 15-year-old hip replacement.
Her radiograph reveals an eccentric position of the ball head within the acetabulum. She has minimal
periacetabular osteolysis, and her uncemented acetabular and femoral components are well fixed. What is
the most appropriate treatment?

1. Revision of the acetabular component with retention of the femoral component


2. Revision of both acetabular and femoral components
3. Liner and ball head exchange
4. Observation

PREFERRED RESPONSE: 3

DISCUSSION
Well-fixed components in the presence of minimal osteolysis and a worn acetabular polyethylene liner in a
symptomatic patient should be treated with liner and ball head exchange. Revision of either or both of the
well-fixed and otherwise well-functioning components is unnecessary and introduces the possibility for
additional complications. Observation is a suboptimal choice considering that significant wear is seen and
the patient is symptomatic.

RECOMMENDED READINGS
Aldinger PR, Jung AW, Pritsch M, Breusch S, Thomsen M, Ewerbeck V, Parsch D. Uncemented grit-
blasted straight tapered titanium stems in patients younger than fifty-five years of age. Fifteen to twenty-
year results. J Bone Joint Surg Am. 2009 Jun;91(6):1432-9. PubMed PMID: 19487522.
Della Valle CJ, Mesko NW, Quigley L, Rosenberg AG, Jacobs JJ, Galante JO. Primary total hip
arthroplasty with a porous-coated acetabular component. A concise follow-up, at a minimum of twenty
years, of previous reports. J Bone Joint Surg Am. 2009 May;91(5):1130-5. PubMed PMID: 19411461.

© 2013 American Academy of Orthopaedic Surgeons 2013 Adult Reconstructive Surgery of the Hip and Knee Self-Assessment Examination
82 • American Academy of Orthopaedic Surgeons

Question 87
When compared to a general population of patients undergoing knee arthroplasty, patients with ankylosing
spondylitis are at higher risk for developing what condition?

1. Stiffness and ossification


2. Infection
3. Component loosening
4. No heightened risk; they can expect the same outcome as patients with primary osteoarthritis

PREFERRED RESPONSE: 1

DISCUSSION
Patients with ankylosing spondylitis experience pain relief and improved function after total knee
replacement, but a number of studies have shown an increased risk for knee stiffness and heterotopic
ossification in this subgroup. Patients should be advised that their underlying condition could make them
more likely to experience this complication.

RECOMMENDED READINGS
Finsterbush A, Amir D, Vatashki E, Husseini N. Joint surgery in severe ankylosing spondylitis. Acta
Orthop Scand. 1988 Oct;59(5):491-6. PubMed PMID: 3188851.
Parvizi J, Duffy GP, Trousdale RT. Total knee arthroplasty in patients with ankylosing spondylitis. J Bone
Joint Surg Am. 2001 Sep;83-A(9):1312-6. PubMed PMID: 11568191.

© 2013 American Academy of Orthopaedic Surgeons 2013 Adult Reconstructive Surgery of the Hip and Knee Self-Assessment Examination
2013 Adult Reconstructive Surgery of the Hip and Knee Self-Assessment Examination Answer Book • 83

Figure 88

CLINICAL SITUATION FOR QUESTIONS 88 AND 89


Figure 88 is the radiograph of an 84-year-old man who had bilateral total knee replacements 14 years
ago. For 3 years he has had pain and swelling in his right knee. Radiographs reveal a progressive valgus
deformity of the knee. The patient wants to return to full function and ambulate throughout his household
with minimal surgical risk.

Question 88
Which of the following descriptions best characterizes the polyethylene particles liberated from this
patient’s knee in comparison to wear particles found in a patient who undergoes total hip arthroplasty?

1. Smaller and less reactive


2. Smaller and more reactive
3. Larger and less reactive
4. Larger and more reactive

PREFERRED RESPONSE: 3

Question 89
What is the most appropriate treatment for this condition?

1. Revision of both the femoral and tibial components


2. Polyethylene exchange
3. Tibia component revision
4. Osteotomy

PREFERRED RESPONSE: 2

© 2013 American Academy of Orthopaedic Surgeons 2013 Adult Reconstructive Surgery of the Hip and Knee Self-Assessment Examination
84 • American Academy of Orthopaedic Surgeons

DISCUSSION FOR QUESTIONS 88 AND 89


The particles isolated from the knee are larger and less reactive. Wear in a total knee replacement
prosthesis differs from that in a total hip replacement. Wear is the result of pitting and delamination; there
are not submicron wear debris particles as one would find in the hip. Because of this patient’s age and
medical risks, polyethylene exchange is the best decision. The components appear to be well-fixed, and
progressive valgus deformity is caused by asymmetric polyethylene wear. Revision total knee arthroplasty
is not expected to be necessary.

RECOMMENDED READINGS FOR QUESTIONS 88 AND 89


Griffin WL. Polyethylene wear and osteolysis in total knee arthroplasty. In: Glassman AH, Lachiewicz
PF, Tanzer M, eds. Orthopaedic Knowledge Update: Hip and Knee Reconstruction 4. Rosemont, IL:
American Academy of Orthopaedic Surgeons; 2011:191-199.
Parvizi J, Bercik M, Albert T. Bearing surface materials for hip, knee, and spinal disk replacement. In:
Flynn JM, ed. Orthopaedic Knowledge Update 10. Rosemont, IL: American Academy of Orthopaedic
Surgeons; 2011:73-83.

END OF SERIES

CLINICAL SITUATION FOR QUESTIONS 90 AND 91


A 76-year-old woman underwent an uncomplicated total knee arthroplasty (TKA) 7 years ago. She
has had a 4-month history of pain and swelling. Radiographs reveal a global large lucency under the
tibial component. Comparison of older radiographs reveals gross migration and subsidence of the tibial
component. Further testing reveals an elevated erythrocyte sedimentation rate and C-reactive protein
level, as well as a synovial white blood cell count > 15000 cells/µL. Synovial fluid cultures are negative
for bacterial growth at 48 hours.

Question 90
What is the most appropriate next step?

1. Synovial fluid gram stain


2. Tissue samples sent for culture and frozen section
3. Revision of both components with antibiotic cement and stems
4. Removal of the tibial component and retention of the femoral component if well fixed and
appropriately positioned

PREFERRED RESPONSE: 2

© 2013 American Academy of Orthopaedic Surgeons 2013 Adult Reconstructive Surgery of the Hip and Knee Self-Assessment Examination
2013 Adult Reconstructive Surgery of the Hip and Knee Self-Assessment Examination Answer Book • 85

Question 91
Frozen sections reveal 17 neutrophils per high-power field. What is the most appropriate definitive
surgical treatment associated with the highest chance for successful eradication of infection?

1. Irrigation and debridement with tibial revision


2. Irrigation and debridement with placement of an intra-articular antibiotic pump
3. Resection arthroplasty with placement of an antibiotic spacer
4. One-stage exchange

PREFERRED RESPONSE: 3

DISCUSSION FOR QUESTIONS 90 AND 91


In this scenario, suspicion for a periprosthetic joint infection is high. The synovial fluid WBC count
is elevated and the synovial fluid culture result may be a false negative. The AAOS Clinical Practice
Guideline, The Diagnosis of Periprosthetic Joint Infections of the Hip and Knee, recommends strongly
against the use of gram stain and strongly in favor of multiple culture and frozen sections to aid in the
diagnosis of infection. Considering the chronicity of the infection, removal of both components is
indicated.

RECOMMENDED READINGS FOR QUESTIONS 90 AND 91


Della Valle C, Parvizi J, Bauer TW, DiCesare PE, Evans RP, Segreti J, Spangehl M, Watters WC 3rd,
Keith M, Turkelson CM, Wies JL, Sluka P, Hitchcock K; American Academy of Orthopaedic Surgeons.
American Academy of Orthopaedic Surgeons clinical practice guideline on: the diagnosis of periprosthetic
joint infections of the hip and knee. J Bone Joint Surg Am. 2011 Jul 20;93(14):1355-7. PubMed PMID:
21792503.
Garvin KL, Konigsberg BS. Infection following total knee arthroplasty: prevention and management. J
Bone Joint Surg Am. 2011 Jun 15;93(12):1167-75. Review. PubMed PMID: 21776555.
Krempec JA, Masonis JL, Fehring TK. Infection in total knee arthroplasty. In: Glassman AH, Lachiewicz
PF, Tanzer M, eds. Orthopaedic Knowledge Update: Hip and Knee Reconstruction 4. Rosemont, IL:
American Academy of Orthopaedic Surgeons; 2011:201-215.

END OF SERIES

© 2013 American Academy of Orthopaedic Surgeons 2013 Adult Reconstructive Surgery of the Hip and Knee Self-Assessment Examination
86 • American Academy of Orthopaedic Surgeons

Question 92
A 68-year-old woman underwent an uncemented medial/lateral tapered femoral placement during a total
hip arthroplasty. The orthopaedic surgeon noticed a nondisplaced vertical fracture in the calcar region of
the femoral neck with final implant insertion. What is the most appropriate treatment?

1. Removal of the press-fit implant and cementing of the same femoral stem
2. Removal of the uncemented femoral component and placement of a revision modular taper-
fluted femoral stem
3. Removal of the implant, placement of a cerclage cable around the femoral neck above the
lesser trochanter, and reinsertion of the implant
4. Final seating of the uncemented femoral component without additional measures

PREFERRED RESPONSE: 3

DISCUSSION
The recognized treatment for a proximal periprosthetic fracture is to first identify the extent and then
optimize the correction. Several studies indicate that proximal cerclage wiring is adequate to create
“barrel hoop” stability of the proximal femur. Braided cables offer superior stability compared to twisted
wires or Luque wires. Finally, the appropriate postoperative treatment is protected weight bearing for 6
weeks, with periodic radiographs taken at 2-week intervals. Other options such as cementing the femoral
stem and using a revision arthroplasty device are indicated for unstable fractures.

RECOMMENDED READINGS
Masri BA, Davidson D, Duncan CP, et al. Total hip arthroplasty complications. In: Barrack RL, Booth RE
Jr, Lonner JH, McCarthy JC, Mont MA, Rubash HE, eds. Orthopaedic Knowledge Update: Hip and Knee
Reconstruction 3. Rosemont, IL: American Academy of Orthopaedic Surgeons; 2006:475-503.
Warren PJ, Thompson P, Fletcher MD. Transfemoral implantation of the Wagner SL stem. The abolition of
subsidence and enhancement of osteotomy union rate using Dall-Miles cables. Arch Orthop Trauma Surg.
2002 Dec;122(9-10):557-60. Epub 2002 Sep 13. PubMed PMID: 12483345.

Question 93
The pharmacokinetics of which deep venous thrombosis (DVT) prophylactic agent are affected by liver
function and dietary intake?

1. Dalteparin
2. Warfarin
3. Fondaparinux
4. Enoxaparin

PREFERRED RESPONSE: 2

© 2013 American Academy of Orthopaedic Surgeons 2013 Adult Reconstructive Surgery of the Hip and Knee Self-Assessment Examination
2013 Adult Reconstructive Surgery of the Hip and Knee Self-Assessment Examination Answer Book • 87

DISCUSSION
Warfarin is an oral vitamin K antagonist that is rapidly absorbed from the gastrointestinal tract. It
accumulates in the liver, where it is metabolized and excreted. The pharmacokinetics of warfarin can be
affected by certain drugs or disease states that influence liver function. Fondaparinux is a synthetic factor
Xa inhibitor that is eliminated through the kidneys. Both Dalteparin and Enoxaparin are low-molecular-
weight heparins that activate antithrombin and inhibit factors Xa and IIa. Like Fondaparinux, they are
eliminated through the kidneys and should be used with caution in patients with kidney disease.

RECOMMENDED READINGS
Colwell CW Jr, Hardwick ME. Venous thromboembolic disease and prophylaxis in total joint arthroplasty.
In: Barrack RL, Booth RE Jr, Lonner JH, McCarthy JC, Mont MA, Rubash HE, eds. Orthopaedic
Knowledge Update: Hip and Knee Reconstruction 3. Rosemont, IL: American Academy of Orthopaedic
Surgeons; 2006:233-240.
Della Valle CJ. Coagulation and thromboembolism. In: Lieberman JR, ed. AAOS Comprehensive
Orthopaedic Review. Vol 1. Rosemont, IL: American Academy of Orthopaedic Surgeons; 2009:149-156.

Figure 94

Question 94
Figure 94 is the radiograph of a patient who underwent component removal, insertion of an antibiotic
spacer, and recent completion of 6 weeks of intravenous antibiotic therapy. The patient’s C-reactive
protein (CRP) and erythrocyte sedimentation rate (ESR) have decreased and are now within defined limits.
The skin is supple and the patient has a range of motion between 10 and 70 degrees. What is the most
appropriate next step?

1. Two weeks off of antibiotics (antibiotic holiday), followed by knee joint aspiration
2. Continued observation for 6 months after surgery to make sure the infection has resolved
3. Joint aspiration for culture and cell count at the time of completion of IV antibiotic therapy
4. Antibiotic spacer exchange

PREFERRED RESPONSE: 1

© 2013 American Academy of Orthopaedic Surgeons 2013 Adult Reconstructive Surgery of the Hip and Knee Self-Assessment Examination
88 • American Academy of Orthopaedic Surgeons

DISCUSSION
The situation described here is consistent with a successful treatment for deep infection. If the patient’s
knee aspiration reveals no organisms and the CRP and ESR trend toward normal, the patient is a candidate
for a second-stage revision knee arthroplasty. A 2-week antibiotic holiday is necessary to obtain accurate
culture results either with aspiration or soft-tissue specimens at the time of second-stage revision TKA.
There is no evidence that waiting for up to 6 months provides a lower recurrence rate, and there may be
long-term problems with a spacer left in place for 6 months, such as breakage, instability, or loosening.

RECOMMENDED READINGS
Sherrell JC, Fehring TK, Odum S, Hansen E, Zmistowski B, Dennos A, Kalore N; Periprosthetic Infection
Consortium. The Chitranjan Ranawat Award: fate of two-stage reimplantation after failed irrigation and
debridement for periprosthetic knee infection. Clin Orthop Relat Res. 2011 Jan;469(1):18-25. PubMed
PMID: 20582495.
Krempec JA, Masonis JL, Fehring TK. Infection in total knee arthroplasty. In: Glassman AH, Lachiewicz
PF, Tanzer M, eds. Orthopaedic Knowledge Update: Hip and Knee Reconstruction 4. Rosemont, IL:
American Academy of Orthopaedic Surgeons; 2011:201-215.

Question 95
What factor is associated with decreased range of motion to impingement?

1. Skirted modular femoral head


2. Trapezoidal neck geometry
3. Vertical cup inclination of 40 to 55 degrees
4. Anteversion of 10 to 20 degrees of both the stem and cup

PREFERRED RESPONSE: 1

DISCUSSION
The use of modular femoral stems and acetabular implants have increased the number of head, neck,
and liner designs, but the features of recent designs can cause intra-articular prosthetic impingement
within the arc of motion required to perform daily activities. Impingement may lead to limited motion,
increased wear, osteolysis, and subluxation or dislocation. Minimizing impingement involves avoiding
skirted heads, matching the femoral head with an appropriate acetabular implant, maximizing the head-
to-neck ratio and, when possible, using a chamfered acetabular liner and a trapezoidal, rather than
circular, neck cross-section. Computer modeling studies indicate the optimal cup position is 45 to 55
degrees’ abduction. Angles < 55 degrees require anteversion of 10 to 20 degrees of both the stem and
cup to minimize risk for impingement and dislocation. However, wear or instability may occur with cup
inclination exceeding 50 degrees.

© 2013 American Academy of Orthopaedic Surgeons 2013 Adult Reconstructive Surgery of the Hip and Knee Self-Assessment Examination
2013 Adult Reconstructive Surgery of the Hip and Knee Self-Assessment Examination Answer Book • 89

RECOMMENDED READINGS
Barrack RL. Dislocation after total hip arthroplasty: implant design and orientation. J Am Acad Orthop
Surg. 2003 Mar-Apr;11(2):89-99. Review. PubMed PMID: 12670135.
Barrack RL, Thornberry RL, Ries MD, Lavernia C, Tozakoglou E. The effect of component design on
range of motion to impingement in total hip arthroplasty. Instr Course Lect. 2001;50:275-80. PubMed
PMID: 11372324.

Question 96
A 68-year-old woman undergoes an uncomplicated total knee replacement through a midline incision that
is extended distally to join a previous incision from a high-tibial osteotomy done 12 years previously.
Despite relief of pain and appropriate knee mobility at 2 weeks, drainage continues from the distal part of
the wound. What are the most appropriate next step(s) in treatment?

1. Oral cephalexin while the wound heals


2. Vacuum suction drain applied over the draining part
3. Intravenous antibiotics and reassess the knee in 24 hours
4. Urgent open debridement of the knee, cultures, and evaluation of inflammatory laboratory data

PREFERRED RESPONSE: 4

DISCUSSION
Drainage from a knee after arthroplasty can be ominous; here, a previous incision was made for an
osteotomy, possibly compromising wound healing. At 2 weeks, persistent wound drainage is not expected
and warrants urgent attention with surgical debridement, cultures, and a baseline check of inflammatory
laboratory data. Intravenous antibiotics can be started while awaiting cultures, but oral or intravenous
antibiotics alone are not sufficient and a vacuum drain is not indicated in this situation.

RECOMMENDED READINGS
Jones RE. Wound healing in total joint arthroplasty. Orthopedics. 2010 Sep 7;33(9):660. doi:
10.3928/01477447-20100722-35. PubMed PMID: 20839686.
Galat DD, McGovern SC, Larson DR, Harrington JR, Hanssen AD, Clarke HD. Surgical treatment
of early wound complications following primary total knee arthroplasty. J Bone Joint Surg Am. 2009
Jan;91(1):48-54. PubMed PMID: 19122078.

© 2013 American Academy of Orthopaedic Surgeons 2013 Adult Reconstructive Surgery of the Hip and Knee Self-Assessment Examination
90 • American Academy of Orthopaedic Surgeons

Question 97
What infection-control measure has been shown to have the most notable impact in reducing surgical-site
infections?

1. Intravenous antibiotic administration within 1 hour of surgical incision


2. Screening and decolonization of patients colonized with methicillin-resistant Staphylococcus
aureus
3. Horizontal laminar flow
4. Use of enclosed body exhaust suits

PREFERRED RESPONSE: 1

DISCUSSION
Timely administration of prophylactic antibiotics is the most important factor shown to decrease surgical-
site infections. The use of horizontal laminar flow and body exhaust suits has not been shown to
significantly affect infection rates.

RECOMMENDED READINGS
Garvin KL, Konigsberg BS. Infection following total knee arthroplasty: prevention and management. J
Bone Joint Surg Am. 2011 Jun 15;93(12):1167-75. Review. PubMed PMID: 21776555.
Tyllianakis ME, Karageorgos ACh, Marangos MN, Saridis AG, Lambiris EE. Antibiotic prophylaxis in
primary hip and knee arthroplasty: comparison between cefuroxime and two specific antistaphylococcal
agents. J Arthroplasty. 2010 Oct;25(7):1078-82. Epub 2010 Apr 8. PubMed PMID: 20381287.

Question 98
Three years after undergoing a metal-on-polyethylene total hip arthroplasty, a 72-year-old woman
develops pain with weight bearing and rest. Hip flexion and internal rotation is associated with pain.
Radiographs show no evidence of loosening. What is the most appropriate next step in this evaluation?

1. Bone scan
2. White blood cell (WBC) count
3. Labeled white cell scan
4. Erythrocyte sedimentation rate (ESR) and C-reactive protein (CRP) levels

PREFERRED RESPONSE: 4

© 2013 American Academy of Orthopaedic Surgeons 2013 Adult Reconstructive Surgery of the Hip and Knee Self-Assessment Examination
2013 Adult Reconstructive Surgery of the Hip and Knee Self-Assessment Examination Answer Book • 91

DISCUSSION
The ESR is a sensitive serologic indicator of inflammation; however, it is less specific for infection. Other
noninfectious conditions may cause an elevated ESR. The CRP is a specific indicator of infection, which
may be transiently elevated after uncomplicated total hip arthroplasty, but usually returns to a normal
range within 1 to 2 weeks of surgery. If both the ESR and CRP are elevated, there is a high likelihood
that a painful hip replacement is infected, and joint aspiration is indicated. Serum WBC is not a reliable
indicator of periprosthetic infection, with a sensitivity of only 20%. Bone scan typically demonstrates
increased technetium uptake for several years after hip arthroplasty, which is not specific for infection. A
labeled white cell scan is useful to rule out infection, but less helpful to detect the presence of infection.

RECOMMENDED READINGS
Parvizi J. Periprosthetic joint infections. In: Lieberman JR, ed. AAOS Comprehensive Orthopaedic
Review. Vol 2. Rosemont, IL: American Academy of Orthopaedic Surgeons; 2009:1067-1073.
Masri BA, Davidson D, Duncan CP, et al. Total hip arthroplasty complications. In: Barrack RL, Booth RE
Jr, Lonner JH, McCarthy JC, Mont MA, Rubash HE, eds. Orthopaedic Knowledge Update: Hip and Knee
Reconstruction 3. Rosemont, IL: American Academy of Orthopaedic Surgeons; 2006:475-503.
Schinsky MF, Della Valle CJ, Sporer SM, Paprosky WG. Perioperative testing for joint infection in
patients undergoing revision total hip arthroplasty. J Bone Joint Surg Am. 2008 Sep;90(9):1869-75.
Erratum in: J Bone Joint Surg Am. 2010 Mar;92(3):707. PubMed PMID: 18762646.

Figure 99a Figure 99b


Question 99
Figure 99a is a CT scan at the level of the distal femur and femoral component. What is the orientation of
the femoral component in the CT scan?

1. Properly rotated
2. Internally rotated
3. Externally rotated
4. Excessive flexion

PREFERRED RESPONSE: 2

© 2013 American Academy of Orthopaedic Surgeons 2013 Adult Reconstructive Surgery of the Hip and Knee Self-Assessment Examination
92 • American Academy of Orthopaedic Surgeons

DISCUSSION
A CT scan with metal artifact reduction is a useful study to evaluate femoral component rotation. Proper
rotation would show that the transepicondylar line and posterior condylar line are parallel. Figure 99b
shows that the femoral component is internally rotated compared to the femoral epicondylar axis.

RECOMMENDED READINGS
Coughlin KM, Incavo SJ, Churchill DL, Beynnon BD. Tibial axis and patellar position relative to the
femoral epicondylar axis during squatting. J Arthroplasty. 2003 Dec;18(8):1048-55. PubMed PMID:
14658111.
Berger RA, Crossett LS, Jacobs JJ, Rubash HE. Malrotation causing patellofemoral complications after
total knee arthroplasty. Clin Orthop Relat Res. 1998 Nov;(356):144-53. PubMed PMID: 9917679.

Question 100
A 68-year-old woman with a 9-year history of type II diabetes is seen 11 weeks after an uncemented left
total hip replacement. When seen 6 weeks after surgery, some mild erythema and induration at the distal
incision was noted, but no drainage. She states that drainage started 2 weeks ago. Examination shows
turbid drainage coming from the distal third of the incision with mild surrounding erythema. Hip range
of motion causes mild discomfort. Investigations reveal an erythrocyte sedimentation rate of 45 mm/h
(reference range, 0-20 mm/h) and C-reactive protein of 54 mg/L (reference range, 0.08-3.1 mg/L). A rapid
polymerase chain reaction of the swabbed fluid is positive for methicillin-resistant Staphylococcus aureus.
Hip aspiration under fluoroscopy is attempted but no fluid is obtained. What is the most appropriate
treatment?

1. Debridement of the skin and superficial tissues


2. Debridement and removal of the implants and insertion of an antibiotic spacer
3. Debridement of superficial and deep tissues including the joint with exchange of the modular
head and liner
4. Prescription for sulfamethoxazole and trimethoprim (Bactrim DS), 1 tablet, twice daily for 14
days, and then re-evaluate the patient

PREFERRED RESPONSE: 2

DISCUSSION
This case illustrates the treatment choices to address a postsurgical deep infection at 11 weeks postsurgery.
Considering the progression of symptoms and persistent drainage, one needs to assume the infection
is deep. Wound drainage beginning at 9 weeks after surgery is unlikely to be the result of a superficial
infection. The absence of fluid on the attempted aspiration may occur in situations in which a sinus tract
allows most of the fluid to escape the joint. Injection of contrast could confirm the presence of a sinus
tract. Debridement and removal of the implants and insertion of an antibiotic spacer are most appropriate
because the results of a single debridement at 11 weeks with a resistant organism are poor for curing
or controlling infection. The use of antibiotics alone or a superficial debridement is inadequate in this
setting.

© 2013 American Academy of Orthopaedic Surgeons 2013 Adult Reconstructive Surgery of the Hip and Knee Self-Assessment Examination
2013 Adult Reconstructive Surgery of the Hip and Knee Self-Assessment Examination Answer Book • 93

RECOMMENDED READINGS
Azzam KA, Seeley M, Ghanem E, Austin MS, Purtill JJ, Parvizi J. Irrigation and debridement in
the management of prosthetic joint infection: traditional indications revisited. J Arthroplasty. 2010
Oct;25(7):1022-7. Epub 2010 Apr 8. PubMed PMID: 20378306.
Jafari SM, Coyle C, Mortazavi SM, Sharkey PF, Parvizi J. Revision hip arthroplasty: infection is the most
common cause of failure. Clin Orthop Relat Res. 2010 Aug;468(8):2046-51. PubMed PMID: 20195808.
Estes CS, Beauchamp CP, Clarke HD, Spangehl MJ. A two-stage retention debridement protocol for
acute periprosthetic joint infections. Clin Orthop Relat Res. 2010 Aug;468(8):2029-38. PubMed PMID:
20224958.
Bradbury T, Fehring TK, Taunton M, Hanssen A, Azzam K, Parvizi J, Odum SM. The fate of acute
methicillin-resistant Staphylococcus aureus periprosthetic knee infections treated by open debridement
and retention of components. J Arthroplasty. 2009 Sep;24(6 Suppl):101-4. Epub 2009 Jun 24. Review.
PubMed PMID: 19553077.

© 2013 American Academy of Orthopaedic Surgeons 2013 Adult Reconstructive Surgery of the Hip and Knee Self-Assessment Examination
Introducing the
AAOS Member Advantage Program

AAOS Revenue Management program


powered by Gateway EDI –
Reduce your
claims rejection rate
within 90 days

AAOS Member
Insurance programs –
Protect your income
and your assets
• Individual
disability income
• Long term care
• Supplemental
term life AAOS Group
Purchasing program
by MedAssets –
Save up to 10% on
supplies and
equipment

Learn more at: www.aaos.org/advantage

Your AAOS membership,


delivering exceptional value to you.

You might also like